Рисунок два конденсатора: Глава 20. Конденсаторы

Содержание

Глава 20. Конденсаторы

Для накопления разноименных электрических зарядов служит устройство, которое называется конденсатором. Конденсатор — система двух изолированных друг от друга проводников (которые часто называют обкладками конденсатора), один из которых заряжен положительным, второй — таким же по величине, но отрицательным зарядом. Если эти проводники представляют собой плоские параллельные пластинки, расположенные на небольшом рас-стоянии друг от друга, то конденсатор называется плоским.

Для характеристики способности конденсатора накапливать заряд вводится понятие электроемкости (часто говорят просто емкости). Емкостью конденсатора называется отношение заряда конденсатора к той разности потенциалов , которая возникает между обкладками при их заряжении зарядами и (эту разность потенциалов проводников часто называют электрическим напряжением между обкладками и обозначают буквой ):

(20.1)

Поскольку величины и (или ) в формуле (20.1) зависимы, то емкость (20.1) не зависит от и , а является характеристикой геометрии системы проводников. Действительно, при сообщении проводникам зарядов и проводники приобретут потенциалы, разность которых будет пропорциональна заряду . Поэтому в отношении (20.1) заряд сокращается.

Выведем формулу для емкости плоского конденсатора (эта формула входит в программу школьного курса физики). При заряжении параллельных пластин, расположенных на небольшом расстоянии друг от друга, зарядами и , в пространстве между ними возникает однородное электрическое поле с напряженностью (см. гл. 18):

(20.2)

Разность потенциалов между пластинами равна

(20.3)

где — площадь пластин, — расстояние между ними. Отсюда, вычисляя отношение заряда к разности потенциалов (20.3), находим емкость плоского конденсатора

(20.4)

Если все пространство между обкладками заполнено диэлектриком с диэлектрической проницаемостью , то поле (20.2) и разность потенциалов (20.3) убывает в раз, а емкость конденсатора в раз взрастает

(20.5)

Для конденсаторов, соединенных в батареи, вводится понятие эквивалентной емкости, как емкости одного конденсатора, который при заряжении его тем же зарядом, что и батарея дает ту же разность потенциалов, что и батарея конденсаторов. Приведем формулы для эквивалентной емкости, а также для заряда и электрического напряжения на каждом конденсаторе при последовательном и параллельном их соединении.

Последовательное соединение (см. рисунок). При сообщении левой пластине левого конденсатора заряда , а правой пластине правого заряда , на внутренних пластинах благодаря поляризации будут индуцироваться заряды (см. рисунок; значения индуцированных зарядов приведены под пластинами). Можно доказать, что в результате поляризации каждый конденсатор будет заряжен такими же зарядами и , как и заряды крайних пластин, напряжение на всей батарее конденсаторов равно сумме напряжений на каждом, а обратная эквивалентная емкость батареи — сумме обратных емкостей всех конденсаторов

(20.6)

Параллельное соединение (см. рисунок). В этом случае если сообщить левому проводнику заряд , правому сообщить заряд , заряд распределится между конденсаторами, вообще говоря, не одинаково, но по закону сохранения заряда .

Поскольку правые пластины всех конденсаторов соединены между собой, левые — тоже, то они представляют собой единые проводники, и, следовательно, разность потенциалов между пластинами каждого конденсатора будет одинакова: . Можно доказать, что при таком соединении конденсаторов эквивалентная емкость батареи равна сумме емкостей отдельных конденсаторов

(20.7)

Заряженный конденсатор обладает определенной энергией. Если конденсатор емкости заряжен зарядом , то энергия этого конденсатора (можно говорить энергия электрического поля конденсатора) равна

(20.8)

С помощью определения электрической емкости (20.1) можно переписать формулу (20.8) еще в двух формах:

(20.9)

Рассмотрим в рамках этого минимума сведений о конденсаторах типичные задачи ЕГЭ по физике, которые были предложены в первой части книги.

Электроемкость конденсатора — его геометрическая характеристика, которая при неизменной геометрии не зависит от заряда конденсатора (задача 20.1.1 — ответ 3). Аналогично не меняется емкость конденсатора при увеличении напряжения на конденсаторе (задача 20.1.2

— ответ 3).

Связь между единицами измерений (задача 20.1.3) следует из определения емкости (20.1). Единица электрической емкости в международной системе единиц измерений СИ называется Фарада. 1 Фарада — это емкость такого конденсатора, между пластинами которого возникает напряжение 1 В при зарядах пластин 1 Кл и -1 Кл (ответ 4).

Поскольку электрическое поле в плоском конденсаторе однородно, то напряженность поля в конденсаторе и напряжение между пластинами связаны соотношением (см. формулу (18.9)) , где — расстояние между пластинами. Отсюда находим напряженность поля между обкладками плоского конденсатора в задаче 20.1.4

(ответ 4).

Согласно определению электрической емкости имеем в задаче 20.1.5

(ответ 2).

Из формулы (20.4) для емкости плоского конденсатора заключаем, что при увеличении площади его пластин в 3 раза (задача 20.1.6) его емкость увеличивается в 3 раза (ответ 1).

При уменьшении в раз расстояния между пластинами емкость плоского конденсатора возрастет в раз. Поэтому новое напряжение на конденсаторе (задача 20.1.7) можно найти из следующей цепочки формул

где и — новый заряд конденсатора (ответ 3).

Так как конденсатор в задаче 20.1.8 подключен к источнику, то между его пластинами поддерживается постоянное напряжение независимо от расстояния между ними. Поэтому заряд конденсатора изменяется при раздвигании пластин так же, как изменяется его емкость. А поскольку при увеличении расстояния между пластинами вдвое емкость конденсатора уменьшается вдвое (см. формулу (20.4)), то вдвое уменьшается и заряд конденсатора (ответ 2).

В задаче 20.1.9 конденсатор отключен от источника в процессе сближения пластин. Поэтому не меняется их заряд. А поскольку напряженность электрического поля между пластинами определяется соотношением (20.2)

то напряженность электрического поля между пластинами также не изменяется (ответ 3). Этот же результат можно получить и через определение емкости с учетом того, что

произведение от расстояния между пластинами не зависит (см. формулу (20.4)).

Из формул (20.8), (20.9) видим, что только одно из приведенных в качестве ответов к задаче 20.1.10 соотношений (а именно — 2) определяет энергию конденсатора.

При последовательном соединении конденсаторов (задача 20.2.1) одинаковыми будут их заряды независимо от значений их электрических емкостей (ответ 2). При параллельном соединении конденсаторов (задача 20.2.2) одинаковыми будут напряжения на каждом из них (ответ

3).

Поскольку конденсатор в задаче 20.2.3 отключен от источ-ника напряжения, его заряд не меняется в процессе раздвигания пластин. Поэтому для исследования изменения энергии конденсатора удобно воспользоваться формулой (20.8)

(1)

Так как при увеличении расстояния между пластинами в раз электрическая емкость конденсатора уменьшается в раз, то согласно формуле (1) энергия конденсатора увеличится в раз (ответ 1).

В задаче 20.2.4 не изменяется напряжение на конденсаторе. Поэтому воспользуемся первой из формул (20.9)

Из этой формулы заключаем, что при увеличении в раз расстояния между пластинами энергия конденсатора уменьшится в раз — ответ 2. (Разница с предыдущей задачей связана с тем, что здесь кроме внешних сил, совершающих работу при раздвигании пластин, совершает работу источник напряжения.)

В задаче 20.2.5 изменяют расстояние между пластинами (и, следовательно, емкость) и заряд конденсатора. Поэтому удобно воспользоваться формулой (20.8)

Из этой формулы заключаем, что при увеличении расстояния между пластинами в 2 раза и увеличении заряда конденсатора в 2 раза его энергия возрастет в 8 раз (ответ

4).

Поскольку в задаче 20.2.6 конденсаторы соединены последовательно, емкость батареи конденсаторов можно найти по формуле (20.6), откуда находим емкость батареи конденсаторов (ответ 2).

В задаче 20.2.7 конденсаторы соединены параллельно, поэтому емкость батареи конденсаторов можно найти по формуле (20.7): (ответ 2).

Основной вопрос, на который нужно ответить в задаче 20.2.8, это как соединены конденсаторы? Последовательно, параллельно, по-другому? Попробуем по-другому расположить в пространстве и изменить длину соединительных проводов, чтобы схема стала более понятной. Очевидно, что можно соединить вершину 1 и вершину 3 («уменьшив» длину провода 1-3), а также вершины 2 и 4. При этом средний конденсатор разворачивается в пространстве, и схема приобретает вид, показанный на рисунке, откуда видно, что конденсаторы соединены параллельно. Поэтому (ответ 1).

Когда в заряженный плоский конденсатор вставляют металлическую пластинку (задача 20.2.9), параллельную обкладкам конденсатора, напряженность электрического поля внутри пластинки становится равным нулю, вне пластинки между обкладками конденсатора остается таким же, каким оно было в отсутствие пластинки , где — заряд конденсатора, — площадь его пластин. Поэтому напряжение между обкладками конденсатора определяется соотношением:

где — расстояние между обкладками конденсатора, — толщина пластинки. Отсюда находим емкость рассматриваемого конденсатора

(ответ 4).

Чтобы найти емкость сферического конденсатора (задача 20.2.10) сообщим его обкладкам заряды и , найдем напряжение между обкладками, вычислим отношение заряда к напряжению. Разность потенциалов двух концентрических сфер, заряженных зарядами и (напряжение между обкладками сферического конденсатора), определена в задаче 19.2.5., откуда находим электрическую емкость сферического конденсатора (ответ 3):

Задачи по электродинамике повышенной сложности

Физика на 100 Электродинамика Гойхман ГС

Задача 1. (Олимпиада «Физтех-2008»). В цепи, показанной на рисунке, ёмкости конденсаторов равны C и 2C. Конденсатор ёмкостью C заряжен до напряжения U0, конденсатор ёмкостью 2C не заряжен. Какое количество теплоты выделится в резисторе после замыкания ключа?

Решение. До замыкания ключа энергия схемы была сосредоточена в конденсаторе С и равна , а заряд на нём равен . После замыкания ключа этот заряд перераспределится между конденсаторами так, что напряжение на них выровняется, то есть . Отсюда . Перетекание заряда (ток) привело не только к перераспределению первоначальной энергии W1 конденсаторами, но и к выделению тепла в резисторе. По закону сохранения энергии W1=W2+Q, где — энергия двух конденсаторов. Отсюда .

Ответ:

Задача 2. В цепи, показанной на рисунке, ёмкость каждого конденсатора равна C. Левый конденсатор заряжен до напряжения U0, а правый до напряжения 3U0. Верхние обкладки конденсаторов имеют противоположные заряды. Найдите U0, если известно, что в резисторе после замыкания ключа выделилось количество теплоты Q.

Решение. До замыкания ключа энергия, запасённая в схеме, равна , а после замыкания — , где U1 — напряжение на конденсаторах, которое найдем, используя закон сохранения заряда. С учётом зарядов противоположного знака до замыкания ключа . Отсюда, . По закону сохранения энергии . Отсюда

или

откуда

.

Ответ:

Задача 3. Источник тока с ЭДС , резистор с большим сопротивлением R и конденсатор ёмкостью C подключены последовательно друг с другом через ключ K (см. рисунок). Вначале ключ разомкнут и конденсатор не заряжен. Найдите количество теплоты, которое выделится в цепи после замыкания ключа в процессе зарядки конденсатора.

Решение. После замыкания ключа на конденсаторе накопится заряд , а энергия будет . При этом источник тока совершит работу . Так как , то по закону сохранения энергии . Отсюда или . И, наконец, .

Ответ:


Задача 4. Конденсатор ёмкостью C, заряженный до напряжения , подключается через резистор с большим сопротивлением R к батарее с ЭДС (см. рисунок). Определите количество теплоты, которое выделится в цепи при зарядке конденсатора до напряжения .

Решение. До замыкания ключа на конденсаторе был накоплен заряд , а энергия — . После замыкания ключа полярность заряда конденсатора осталась прежней, но заряд увеличился при энергии . Источник тока при этом совершил работу . По закону сохранения энергии

.

Отсюда

.

И, наконец,

.

Ответ:

Задача 5. Конденсатор ёмкостью C, заряженный до напряжения , разряжается через резистор с большим сопротивлением R и батарею с ЭДС (см. рисунок). Найдите количество теплоты, выделившейся при разрядке конденсатора.

Решение. До замыкания ключа на конденсаторе был накоплен заряд , а энергия — . После замыкания ключа полярность заряда конденсатора осталась прежней, но заряд уменьшился при энергии . Источник тока при этом совершил работу . По закону сохранения энергии

.

Отсюда

.

И, наконец,

.

Ответ:

Задача 6 (Олимпиада «Физтех-2015»). В электрической цепи, схема которой показана на рисунке, все элементы идеальные, их параметры указаны. До замыкания ключа ток в цепи отсутствовал. Ключ на некоторое время замыкают, а затем размыкают. За время, пока ключ был замкнут, через резистор 2R протек заряд q0 . После размыкания ключа через тот же резистор протек заряд 2q0 .

  1. Найдите ток через источник сразу после замыкания ключа.

  2. Найдите количество теплоты, которое выделилось в цепи после размыкания ключа.

  3. Найдите количество теплоты, которое выделилось в цепи при замкнутом ключе.

Решение. При решении задач, подобных этой надо понимать, что между пластинами (обкладками) конденсатора ток течь не может (там хороший диэлектрик). Напротив, если на подводящих проводах создать разность потенциалов, то электроны, как носители электрического заряда в металлических проводниках, придут в движение. При этом на пластинах накапливаются заряды противоположного знака. Происходит это не мгновенно, а с течением времени, зависящего от ёмкости конденсатора и сопротивления резисторов в подводящих цепях. В схеме на рисунке изначально, судя по условию задачи, конденсатор не заряжен. Потенциалы его пластин одинаковы и равны нулю. Поэтому сразу после замыкания ключа тока через резистор 2R нет, так как напряжение на нём, как и на конденсаторе, равно нулю. Ясно, что ток через источник после замыкания ключа равен . Так как после размыкания ключа через резистор 2R протекает заряд 2q0, то именно этот заряд и был накоплен на конденсаторе, пока ключ был замкнут. Следовательно, на конденсаторе накопленная энергия равна , которая после размыкания ключа выделится в виде теплоты. С другой стороны, пока ключ был замкнут через резистор 2R протёк заряд q0. Таким образом, при замкнутом ключе из источника вытек заряд 2q0+q0=3q0, и по закону сохранения энергии работа сторонних сил источника тока равна накопленной на конденсаторе энергии теплу, выделившемуся на обоих резисторах, то есть . Отсюда .

Ответ: , ,

Задача 7 (Олимпиада «Физтех-2015»). В электрической цепи, схема которой показана на рисунке, все элементы идеальные, их параметры указаны. До замыкания ключа ток в цепи отсутствовал. Ключ на некоторое время замыкают, а затем размыкают. Сразу после замыкания ключа ток через резистор 2R равен I0 . Сразу после размыкания ключа ток через этот же резистор равен 2I0 .

1) Найдите количество теплоты, которое выделится в цепи после размыкания ключа.

2) Найдите ток, текущий через источник непосредственно перед размыканием ключа.

3) Найдите заряд, протекший через резистор 2R при замкнутом ключе.

Решение. При решении задач подобного типа надо знать, что «идеальность» катушки означает, что сопротивление её проводов пренебрежимо мало по сравнению с сопротивлением резисторов на схеме. Решающим является также тот факт, что при замыкании-размыкании ключа сила тока через катушку некоторое время (пусть и небольшое) будет изменяться, несмотря на питание источником постоянного тока. Это связано с явлением самоиндукции. В данном случае при замыкании ключа ток в катушке нарастает постепенно, а при размыкании ключа ток уменьшается также постепенно. Итак, если сразу после размыкания ключа ток через катушку равен 2I0, то непосредственно перед этим ток в катушке был также 2I0. Значит, в катушке к этому моменту времени была накоплена энергия магнитного поля .Эта энергия и выделится в виде тепла в цепи после размыкания ключа. Cразу после замыкания ключа ток через катушку отсутствует. Это означает, что из источника вытекает ток I0. Следовательно, ЭДС индукции источника тока . Для контура, состоящего из источника, резисторов R и 2R, закон Ома для момента «перед размыканием» запишется в виде . С учётом найденного значения ЭДС имеем . Отсюда , далее . И, наконец, . Теперь рассмотрим контур, содержащий катушку L и резистор 2R. Закон Ома для этого контура запишется в виде . Здесь необходимо пояснение. Справа стоит нуль, так как в контуре отсутствует источник ЭДС. Слева первое слагаемое — это падение напряжения на резисторе. Второе слагаемое — падение напряжения на катушке. Почему в таком виде? Да потому что сопротивление катушки равно нулю (см. первый абзац) и ЭДС самоиндукции (ток меняется!) компенсирует падение напряжения (знак «минус» в скобках). Преобразуем это выражение . За всё время пока ключ был замкнут изменение тока в катушке , а равно заряду q, протекшему за это время через резистор 2R. Поэтому .

Ответ:

С

L

+

Задача 8 (Олимпиада «Физтех-2014»). В схеме, показанной на рисунке, все элементы можно считать идеальными, параметры элементов указаны на рисунке. До замыкания ключа конденсатор был заряжен до напряжения . Ключ замыкают.

  1. Найдите максимальный ток в цепи.

  2. Найдите ток в момент, когда заряд на конденсаторе равен нулю

Решение. При замыкании ключа в контуре начинается колебательный процесс. Так как ЭДС индукции в катушке пропорциональна скорости изменения силы тока, то при максимальном токе напряжение на катушке равно нулю. Значит для ответа на первый вопрос учтём, что на конденсаторе в этот момент будет напряжение . Если до замыкания ключа на левой пластине был заряд , то после замыкания через некоторое время на левой пластине заряд будет . Поэтому работа сторонних сил за это время равна . Энергия до замыкания ключа была сконцентрирована в конденсаторе , а после — в конденсаторе и в катушке . По закону сохранения энергии . Отсюда . После упрощения получим и, наконец, .

Для ответа на второй вопрос будем иметь в виду, что энергии в конденсаторе нет, а в катушке равна . Опять же по закону сохранения энергии имеем . В данном случае . Тогда . Отсюда . И, наконец, .

Ответ: ;

Задача 9 (ЕГЭ-2012) Источник постоянного напряжения с ЭДС 100 В подключён через резистор к конденсатору, расстояние между пластинами которого можно изменять (см. рисунок). Пластины раздвинули, совершив при этом работу 90 мкДж против сил притяжения пластин. На какую величину изменилась ёмкость конденсатора, если за время движения пластин на резисторе выделилось количество теплоты 40 мкДж? Потерями на излучение пренебречь.

Решение. Вначале энергия конденсатора равна , а после того, как пластины раздвинули, стала равна . Понятно, что ёмкость уменьшается. При этом была совершена работа A внешними силами против сил притяжения пластин и работа сторонних сил в источнике тока , так как в процессе изменения ёмкости в источнике протёк заряд . По закону сохранения энергии или . С учётом того, что , а , получим или .

Откуда . И, наконец,

Ответ:

Задача 10 (Олимпиада «Физтех-2002»).

  Плоский конденсатор, квадратные пластины которого имеют площадь S и расположены на расстоянии d, полностью заполнен твердым диэлектриком с диэлектрической проницаемостью ε (см. рис.). Конденсатор подсоединен к батарее, ЭДС которой равна Диэлектрическую пластину выдвигают из конденсатора. На какое расстояние х выдвинута пластина, если при этом внешними силами совершена работа А? Внутренним сопротивлением батареи пренебречь.

Решение. Для первоначального состояния

, , .

Во втором случае конденсатор становится составным: одна часть заполнена диэлектриком, а вторая — нет, причём соединены они параллельно. Поэтому (пластины квадратные!)

,

,

.

Изменение заряда на конденсаторе составит

.

По закону сохранения энергии . Отсюда

.

Опуская подробности алгебраических преобразований, получим

.

Отсюда

Задачи для самостоятельного решения.

  1. (Олимпиада «Физтех-2015»). В электрической цепи, схема которой показана на рисунке, все элементы идеальные, их параметры указаны. До замыкания ключа ток в цепи отсутствовал. Ключ на некоторое время замыкают, а затем размыкают. Сразу после замыкания ключа ток через резистор 3R равен I0. Сразу после размыкания ключа ток через этот же резистор равен 3I0 .

  1. Найдите количество теплоты, которое выделится в цепи после размыкания ключа.

  2. Найдите ток, текущий через источник непосредственно перед размыканием ключа.

  3. Найдите заряд, протекший через резистор 3R при замкнутом ключе.

  1. (Олимпиада «Физтех-2015»). На рисунке показана схема электрической цепи. Все элементы идеальные, их параметры указаны. До замыкания ключа ток в цепи отсутствовал. Ключ на некоторое время замыкают, а затем размыкают. Сразу после замыкания ключа ток через источник равен I0 . Сразу после размыкания ключа ток через резистор R равен 0,5I0.

  1. Найдите количество теплоты, которое выделится в цепи после размыкания ключа.

  2. Найдите ток, текущий через источник непосредственно перед размыканием ключа.

  3. Найдите заряд, протекший через резистор R при замкнутом ключе.

  1. (Олимпиада «Физтех-2015»). В электрической цепи, схема которой показана на рисунке, все элементы идеальные, их параметры указаны. До замыкания ключа ток в цепи отсутствовал. Ключ на некоторое время замыкают, а затем размыкают. За время, пока ключ был замкнут, через резистор 3R протек некоторый заряд. После размыкания ключа через тот же резистор протек заряд в 2 раза больший. При этом после размыкания ключа в цепи выделилось количество теплоты Q1.

  1. Найдите ток через источник сразу после замыкания ключа.

  2. Найдите заряд, протекший через конденсатор при замкнутом ключе.

  3. Найдите количество теплоты, которое выделилось в цепи при замкнутом ключе.


  1. (Олимпиада «Физтех-2015»). В электрической цени, схема которой показана на рисунке, все элементы идеальные, их параметры указаны. До замыкания ключа ток в цени отсутствовал. Ключ на некоторое время замыкают, а затем размыкают. За время, пока ключ был замкнут, через резистор R протек заряд q0. После размыкания ключа через тот же резистор протек заряд q0/2.

  1. Найдите ток через источник сразу после замыкания ключа.

  2. Найдите количество теплоты, которое выделилось в цепи после размыкания ключа.

  3. Найдите количество теплоты, которое выделилось в цепи при замкнутом ключе.

С

L

+

(Олимпиада «Физтех-2014»). В схеме, показанной на рисунке, все элементы можно считать идеальными, параметры элементов указаны на рисунке. До замыкания ключа конденсатор был заряжен до напряжения . Ключ замыкают.

  1. Найдите максимальный ток в цепи.

  2. Найдите ток в момент, когда заряд на конденсаторе равен нулю.

С

L

+

I

(Олимпиада «Физтех-2014»). В схеме, показанной на рисунке, все элементы можно считать идеальными, параметры элементов указаны на рисунке. До замыкания ключа конденсатор был заряжен до напряжения 3, а в катушке шёл ток слева направо. Ключ замыкают.

  1. Найдите максимальный ток в цепи.

  2. Найдите ток в момент, когда заряд на конденсаторе равен нулю.

  1. (Олимпиада «Физтех-2008»). В цепи, показанной на рисунке, ёмкость каждого конденсатора равна C. Левый конденсатор заряжен до напряжения U0, а правый до напряжения 3U0. У обоих конденсаторов положительный заряд находится на верхней обкладке. Найдите U0, если известно, что в резисторе после замыкания ключа выделилось количество теплоты Q.

+

Источник тока с ЭДС , резистор с большим сопротивлением R и конденсатор ёмкостью C подключены последовательно друг с другом через ключ K (см. рисунок). Вначале ключ разомкнут, а конденсатор заряжен до напряжения . Найдите количество теплоты, которое выделится в цепи после замыкания ключа в процессе зарядки конденсатора.

  1. (Олимпиада «Физтех-2002»). Плоский конденсатор, пластины которого имеют площадь S и расположены на расстоянии d, заполнен твердым диэлектриком с диэлектрической проницаемостью ε (см. рис.). Конденсатор подсоединен к батарее постоянного тока, ЭДС которой равна . Правую пластину конденсатора отодвигают так, что образуется воздушный зазор. На какое расстояние x отодвинута пластина, если при этом внешними силами была совершена работа А? Внутренним сопротивлением батареи пренебречь.

С

L

(Олимпиада «Физтех-2002»). Батарею с ЭДС подключают к последовательно соединенным катушке с индуктивностью L и незаряженному конденсатору емкостью С. В контуре происходят колебания тока. В тот момент, когда ток в контуре становится равным нулю, батарею отключают от схемы и подключают вновь, поменяв местами ее выводы. Чему будет равен после этого максимальный ток в контуре? Внутренним сопротивлением батареи и сопротивлением катушки пренебречь.

Ответ: ;

Все о конденсаторах

Конденсатор — двухполюсник с постоянным или переменным значением ёмкости и малой проводимостью; устройство для накопления заряда и энергии электрического поля.

Во всех радиотехнических и электронных устройствах кроме транзисторов и микросхем применяются конденсаторы. В одних схемах их больше, в других меньше, но совсем без конденсаторов не бывает практически ни одной электронной схемы.

При этом конденсаторы могут выполнять в устройствах самые разные задачи. Прежде всего, это емкости в фильтрах выпрямителей и стабилизаторов. С помощью конденсаторов передается сигнал между усилительными каскадами, строятся фильтры низких и высоких частот, задаются временные интервалы в выдержках времени и подбирается частота колебаний в различных генераторах.

Свою родословную конденсаторы ведут от лейденской банки, которую в середине XVIII века в своих опытах использовал голландский ученый Питер ван Мушенбрук. Жил он в городе Лейдене, так что нетрудно догадаться, почему так называлась эта банка.

Собственно это и была обыкновенная стеклянная банка, выложенная внутри и снаружи оловянной фольгой – станиолем. Использовалась она в тех же целях, как и современная алюминиевая, но тогда алюминий открыт еще не был.

Единственным источником электричества в те времена была электрофорная машина, способная развивать напряжение до нескольких сотен киловольт. Вот от нее и заряжали лейденскую банку. В учебниках физики описан случай, когда Мушенбрук разрядил свою банку через цепь из десяти гвардейцев взявшихся за руки.

В то время никто не знал, что последствия могут быть трагическими. Удар получился достаточно чувствительным, но не смертельным. До этого не дошло, ведь емкость лейденской банки была незначительной, импульс получился очень кратковременным, поэтому мощность разряда была невелика.

Как устроен конденсатор

Устройство конденсатора практически ничем не отличается от лейденской банки: все те же две обкладки, разделенные диэлектриком. Именно так на современных электрических схемах изображаются конденсаторы. На рисунке 1 показано схематичное устройство плоского конденсатора и формула для его расчета.

Рисунок 1. Устройство плоского конденсатора

Здесь S – площадь пластин в квадратных метрах, d – расстояние между пластинами в метрах, C — емкость в фарадах, ε – диэлектрическая проницаемость среды. Все величины, входящие в формулу, указаны в системе СИ. Эта формула справедлива для простейшего плоского конденсатора: можно просто расположить рядом две металлические пластины, от которых сделаны выводы. Диэлектриком может служить воздух.

Из этой формулы можно понять, что емкость конденсатора тем больше, чем больше площадь пластин и чем меньше расстояние между ними. Для конденсаторов с другой геометрией формула может быть иной, например, для емкости одиночного проводника или электрического кабеля. Но зависимость емкости от площади пластин и расстояния между ними та же, что и у плоского конденсатора: чем больше площадь и чем меньше расстояние, тем больше емкость.

На самом деле пластины не всегда делаются плоскими. У многих конденсаторов, например металлобумажных, обкладки представляют собой алюминиевую фольгу свернутую вместе с бумажным диэлектриком в плотный клубок, по форме металлического корпуса.

Для увеличения электрической прочности тонкая конденсаторная бумага пропитывается изолирующими составами, чаще всего трансформаторным маслом. Такая конструкция позволяет делать конденсаторы с емкостью до нескольких сотен микрофарад. Примерно так же устроены конденсаторы и с другими диэлектриками.

Формула не содержит никаких ограничений на площадь пластин S и расстояние между пластинами d. Если предположить, что пластины можно развести очень далеко, и при этом площадь пластин сделать совсем незначительной, то какая-то емкость, пусть небольшая, все равно останется. Подобное рассуждение говорит о том, что даже просто два проводника, расположенные по соседству, обладают электрической емкостью.

Этим обстоятельством широко пользуются в высокочастотной технике: в некоторых случаях конденсаторы делаются просто в виде дорожек печатного монтажа, а то и просто двух скрученных вместе проводков в полиэтиленовой изоляции. Обычный провод–лапша или кабель также обладают емкостью, причем с увеличением длины она увеличивается.

Кроме емкости C, любой кабель обладает еще и сопротивлением R. Оба этих физических свойства распределены по длине кабеля, и при передаче импульсных сигналов работают как интегрирующая RC – цепочка, показанная на рисунке 2.

Рисунок 2.

На рисунке все просто: вот схема, вот входной сигнал, а вот он же на выходе. Импульс искажается до неузнаваемости, но это сделано специально, для чего и собрана схема. Пока же речь идет о влиянии емкости кабеля на импульсный сигнал. Вместо импульса на другом конце кабеля появится вот такой «колокол», а если импульс короткий, то он может и вовсе не дойти до другого конца кабеля, вовсе пропасть.

Исторический факт

Здесь вполне уместно вспомнить историю о том, как прокладывали трансатлантический кабель. Первая попытка в 1857 году потерпела неудачу: телеграфные точки – тире (прямоугольные импульсы) искажались так, что на другом конце линии длиной 4000 км разобрать ничего не удалось.

Вторая попытка была предпринята в 1865 году. К этому времени английский физик У. Томпсон разработал теорию передачи данных по длинным линиям. В свете этой теории прокладка кабеля оказалась более удачной, сигналы принять удалось.

За этот научный подвиг королева Виктория пожаловала ученого рыцарством и титулом лорда Кельвина. Именно так назывался небольшой город на побережье Ирландии, где начиналась прокладка кабеля. Но это просто к слову, а теперь вернемся к последней букве в формуле, а именно, к диэлектрической проницаемости среды ε.

Немножко о диэлектриках

Эта ε стоит в знаменателе формулы, следовательно, ее увеличение повлечет за собой возрастание емкости. Для большинства используемых диэлектриков, таких как воздух, лавсан, полиэтилен, фторопласт эта константа практически такая же, как у вакуума. Но вместе с тем существует много веществ, диэлектрическая проницаемость которых намного выше. Если воздушный конденсатор залить ацетоном или спиртом, то его емкость возрастет раз в 15…20.

Но подобные вещества обладают кроме высокой ε еще и достаточно высокой проводимостью, поэтому такой конденсатор заряд держать будет плохо, он быстро разрядится сам через себя. Это вредное явление называется током утечки. Поэтому для диэлектриков разрабатываются специальные материалы, которые позволяют при высокой удельной емкости конденсаторов обеспечивать приемлемые токи утечки. Именно этим и объясняется такое разнообразие видов и типов конденсаторов, каждый из которых предназначен для конкретных условий.

Электролитический конденсатор

Наибольшей удельной емкостью (соотношение емкость / объем) обладают электролитические конденсаторы. Емкость «электролитов» достигает до 100 000 мкФ, рабочее напряжение до 600В. Такие конденсаторы работают хорошо только на низких частотах, чаще всего в фильтрах источников питания. Электролитические конденсаторы включаются с соблюдением полярности.

Электродами в таких конденсаторах является тонкая пленка из оксида металлов, поэтому часто эти конденсаторы называют оксидными. Тонкий слой воздуха между такими электродами не очень надежный изолятор, поэтому между оксидными обкладками вводится слой электролита. Чаще всего это концентрированные растворы кислот или щелочей.

На рисунке 3 показан один из таких конденсаторов.

Рисунок 3. Электролитический конденсатор

Чтобы оценить размеры конденсатора рядом с ним сфотографировался простой спичечный коробок. Кроме достаточно большой емкости на рисунке можно разглядеть еще и допуск в процентах: ни много ни мало 70% от номинальной.

В те времена, когда компьютеры были большими и назывались ЭВМ, такие конденсаторы стояли в дисководах (по-современному HDD). Информационная емкость таких накопителей теперь может вызвать лишь улыбку: на двух дисках диаметром 350 мм хранилось 5 мегабайт информации, а само устройство весило 54 кг.

Основным назначением показанных на рисунке суперконденсаторов был вывод магнитных головок из рабочей зоны диска при внезапном отключении электроэнергии. Такие конденсаторы могли хранить заряд несколько лет, что было проверено на практике.

Чуть ниже с электролитическими конденсаторами будет предложено проделать несколько простых опытов, чтобы понять, что может делать конденсатор.

Для работы в цепях переменного тока выпускаются неполярные электролитические конденсаторы, вот только достать их почему-то очень непросто. Чтобы как-то эту проблему обойти, обычные полярные «электролиты» включают встречно-последовательно: плюс-минус-минус-плюс.

Если полярный электролитический конденсатор включить в цепь переменного тока, то сначала он будет греться, а потом раздастся взрыв. Отечественные старые конденсаторы разлетались во все стороны, импортные же имеют специальное приспособление, позволяющее избежать громких выстрелов. Это, как правило, либо крестовая насечка на донышке конденсатора, либо отверстие с резиновой пробкой, расположенное там же.

Очень не любят электролитические конденсаторы повышенного напряжения, даже если полярность соблюдена. Поэтому никогда не надо ставить «электролиты» в цепь, где предвидится напряжение близкое к максимальному для данного конденсатора.

Иногда в некоторых, даже солидных форумах, начинающие задают вопрос: «На схеме означен конденсатор 470µF * 16V, а у меня есть 470µF * 50V, можно ли его поставить?». Да, конечно можно, вот обратная замена недопустима.

Конденсатор может накапливать энергию

Разобраться с этим утверждением поможет простая схема, показанная на рисунке 4.

Рисунок 4. Схема с конденсатором

Главным действующим лицом этой схемы является электролитический конденсатор C достаточно большой емкости, чтобы процессы заряда – разряда протекали медленно, и даже очень наглядно. Это дает возможность наблюдать работу схемы визуально с помощью обычной лампочки от карманного фонаря. Фонари эти давно уступили место современным светодиодным, но лампочки для них продаются до сих пор. Поэтому, собрать схему и провести простые опыты очень даже просто.

Может быть, кто-то скажет: «А зачем? Ведь и так все очевидно, да если еще и описание почитать…». Возразить тут, вроде, нечего, но любая, даже самая простая вещь остается в голове надолго, если ее понимание пришло через руки.

Итак, схема собрана. Как она работает?

В положении переключателя SA, показанном на схеме, конденсатор C заряжается от источника питания GB через резистор R по цепи: +GB __ R __ SA __ C __ -GB. Зарядный ток на схеме показан стрелкой с индексом iз. Процесс заряда конденсатора показан на рисунке 5.

Рисунок 5. Процесс заряда конденсатора

На рисунке видно, что напряжение на конденсаторе возрастает по кривой линии, в математике называемой экспонентой. Ток заряда прямо-таки зеркально отражает напряжение заряда. По мере того, как напряжение на конденсаторе растет, ток заряда становится все меньше. И только в начальный момент соответствует формуле, показанной на рисунке.

Через некоторое время конденсатор зарядится от 0В до напряжения источника питания, в нашей схеме до 4,5В. Весь вопрос в том, как это время определить, сколько ждать, когда же конденсатор зарядится?

Постоянная времени «тау» τ = R*C

В этой формуле просто перемножаются сопротивление и емкость последовательно соединенных резистора и конденсатора. Если, не пренебрегая системой СИ, подставить сопротивление в Омах, емкость в Фарадах, то результат получится в секундах. Именно это время необходимо для того, чтобы конденсатор зарядился до 36,8% напряжения источника питания. Соответственно для заряда практически до 100% потребуется время 5* τ.

Часто, пренебрегая системой СИ, подставляют в формулу сопротивление в Омах, а емкость в микрофарадах, тогда время получится в микросекундах. В нашем случае результат удобнее получить в секундах, для чего придется микросекунды просто умножить на миллион, а проще говоря, переместить запятую на шесть знаков влево.

Для схемы, показанной на рисунке 4, при емкости конденсатора 2000мкФ и сопротивлении резистора 500Ω постоянная времени получится τ = R*C = 500 * 2000 = 1000000 микросекунд или ровно одна секунда. Таким образом, придется подождать приблизительно 5 секунд, пока конденсатор зарядится полностью.

Если по истечении указанного времени переключатель SA перевести в правое положение, то конденсатор C разрядится через лампочку EL. В этот момент получится короткая вспышка, конденсатор разрядится и лампочка погаснет. Направление разряда конденсатора показано стрелкой с индексом iр. Время разряда также определяется постоянной времени τ. График разряда показан на рисунке 6.

Рисунок 6. График разряда конденсатора

Конденсатор не пропускает постоянный ток

Убедиться в этом утверждении поможет еще более простая схема, показанная на рисунке 7.

Рисунок 7. Схема с конденсатором в цепи постоянного тока

Если замкнуть переключатель SA, то последует кратковременная вспышка лампочки, что свидетельствует о том, что конденсатор C зарядился через лампочку. Здесь же показан и график заряда: в момент замыкания переключателя ток максимальный, по мере заряда конденсатора уменьшается, а через некоторое время прекращается совсем.

Если конденсатор хорошего качества, т.е. с малым током утечки (саморазряда) повторное замыкание выключателя к вспышке не приведет. Для получения еще одной вспышки конденсатор придется разрядить.

Конденсатор в фильтрах питания

Конденсатор ставится, как правило, после выпрямителя. Чаще всего выпрямители делаются двухполупериодными. Наиболее распространенные схемы выпрямителей показаны на рисунке 8.

Рисунок 8. Схемы выпрямителей

Однополупериодные выпрямители также применяются достаточно часто, как правило, в тех случаях, когда мощность нагрузки незначительна. Самым ценным качеством таких выпрямителей является простота: всего один диод и обмотка трансформатора.

Для двухполупериодного выпрямителя емкость конденсатора фильтра можно рассчитать по формуле

C = 1000000 * Po / 2*U*f*dU, где C емкость конденсатора мкФ, Po мощность нагрузки Вт, U напряжение на выходе выпрямителя В, f частота переменного напряжения Гц, dU амплитуда пульсаций В.

Большое число в числителе 1000000 переводит емкость конденсатора из системных Фарад в микрофарады. Двойка в знаменателе представляет собой число полупериодов выпрямителя: для однополупериодного на ее месте появится единица

C = 1000000 * Po / U*f*dU,

а для трехфазного выпрямителя формула примет вид C = 1000000 * Po / 3*U*f*dU.

Суперконденсатор – ионистор

В последнее время появился новый класс электролитических конденсаторов, так называемый ионистор. По своим свойствам он похож на аккумулятор, правда, с несколькими ограничениями.

Заряд ионистора до номинального напряжения происходит в течение короткого времени, буквально за несколько минут, поэтому его целесообразно использовать в качестве резервного источника питания. По сути ионистор прибор неполярный, единственное, чем определяется его полярность это зарядкой на заводе – изготовителе. Чтобы в дальнейшем эту полярность не перепутать она указывается знаком +.

Большую роль играют условия эксплуатации ионисторов. При температуре 70˚C при напряжении 0,8 от номинального гарантированная долговечность не более 500 часов. Если же прибор будет работать при напряжении 0,6 от номинального, а температура не превысит 40 градусов, то исправная работа возможна в течение 40 000 часов и более.

Наиболее распространенное применение ионистора это источники резервного питания. В основном это микросхемы памяти или электронные часы. В этом случае основным параметром ионистора является малый ток утечки, его саморазряд.

Достаточно перспективным является использование ионисторов совместно с солнечными батареями. Здесь также сказывается некритичность к условию заряда и практически неограниченное число циклов заряд-разряд. Еще одно ценное свойство в том, что ионистор не нуждается в обслуживании.

Пока получилось рассказать, как и где работают электролитические конденсаторы, причем, в основном в цепях постоянного тока.

Ранее ЭлектроВести писали, что в новом исследовании ученые создали микропленочный ультратонкий конденсатор для накопления энергии, который может приклеиваться к поверхности как стикер. Батарея прикрепляется с помощью ультракоротких лазерных импульсов, которые частично расплавляют ее, позволяя удерживаться почти на любой поверхности.

По материалам: electrik.info.

Электрическая емкость (страница 1)

Решение:
При перемещении пластины емкость конденсатора в данный момент времени определяется той частью площади пластин, по которой они перекрывают друг друга. В моменты времени t1 и t2 площади

где l=10 см-длина стороны пластины. В эти моменты времени конденсатор имеет емкости

а заряды на его пластинах

11 Найти заряд, который нужно сообщить двум параллельно соединенным конденсаторам с емкостями C1 = 2 мкФ и С2=1 мкФ, чтобы зарядить их до разности потенциалов V=20кВ.

Решение:
Общий заряд параллельно соединенных конденсаторов

12 Два одинаковых плоских конденсатора соединены параллельно и заряжены до разности потенциалов V0 = 6 В. Найти разность потенциалов V между пластинами конденсаторов, если после отключения конденсаторов от источника тока у одного конденсатора уменьшили расстояние между пластинами вдвое.

Решение:

13 Два конденсатора с емкостями С1 = 1 мкФ и С2 = 2мкФ зарядили до разностей потенциалов V1=20B и V2 = 50 В. Найти разность потенциалов V после соединения — конденсаторов одноименными полосами.

Решение:



14 Конденсатор емкости C1 = 20 мкФ, заряженный до разности потенциалов V1 = 100B, соединили параллельно с заряженным до разности потенциалов V1=40 В конденсатором, емкость которого С2 неизвестна (соединили одноименно заряженные обкладки конденсаторов). Найти емкость С2 второго конденсатора, если разность потенциалов между обкладками конденсаторов после соединения оказалась равной V=80 В.

Решение:

15 Конденсатор емкости С1=4мкФ, заряженный до разности потенциалов V1 = 10B, соединен параллельно с заряженным до разности потенциалов V2 = 20 В конденсатором емкости С2 = 6 мкФ (соединили разноименно заряженные обкладки конденсаторов). Какой заряд окажется на пластинах первого конденсатора после соединения?

Решение:
Заряды конденсаторов до их соединения q1 = C1V1 и q2 = C2V2. После соединения разноименно заряженных обкладок конденсаторов общий заряд q = |q2-q1| = (C1 + C2)V и заряд первого конденсатора где V-разность потенциалов между обкладками конденсаторов после соединения; отсюда

16 Конденсатор, заряженный до разности потенциалов V1 = 20 В, соединили параллельно с заряженным до разности потенциалов V2 = 4 В конденсатором емкости С2 = 33 мкФ (соединили разноименно заряженные обкладки конденсаторов). Найти емкость С1 первого конденсатора, если разность потенциалов между обкладками конденсаторов после их соединения V=2 В.

Решение:
После соединения разноименных обкладок общий заряд q = CV равен разности зарядов q1 = C1V1 и q2 = C2V2 отдельных конденсаторов, где С=С1 + С2 — общая емкость после соединения. Таким образом,


17 Конденсатор емкости С1 = 1 мкФ, заряженный до разности потенциалов V1 = 100B, соединили с конденсатором емкости С2 = 2 мкФ, разность потенциалов V2 на обкладках которого неизвестна (соединили разноименно заряженные обкладки конденсаторов). Найти разность потенциалов V2, если разность потенциалов между обкладками конденсаторов после соединения оказалась равной V=200 В.

Решение:
До соединения заряды первого и второго конденсаторов

После соединения разноименных обкладок общий заряд

Двойной знак мы здесь поставили потому, что заранее не известно, какой из зарядов, q2 или q1 больше; отсюда

Решение со знаком минус соответствует случаю, когда знаки зарядов на пластинах первого конденсатора после соединения пластин не меняются, а со знаком плюс-случаю, когда эти знаки становятся обратными. Так как в нашем случае , а величина |V2| должна быть всегда положительной, то существует лишь одно решение-со знаком плюс. В результате |V2| = 350 В.

18 Два проводящих шара с радиусами R1 и R2 расположены так, что расстояние между ними во много раз больше радиуса большего шара. На шар радиуса R1 помещен заряд q. Каковы будут заряды на шарах после соединения их проводником, если второй шар не был заряжен? Емкостью проводника, соединяющего шары, пренебречь.

Решение:


19 Два проводящих шара с радиусами R1 = 8см и R2 = 20 см, находящихся на большом расстоянии друг от друга, имели электрические заряды q1=40 нКл и q2=— 20 нКл. Как перераспределятся заряды, если шары соединить проводником? Емкостью проводника, соединяющего шары, пренебречь.

Решение:
Соединение шаров проводником эквивалентно параллельному соединению конденсаторов. После соединения


20 Два проводящих шара с радиусами R1 = 10см и R2 = 5см, заряженных до потенциалов φ1=20B и φ2=10В, соединяются проводником. Найти поверхностные плотности зарядов на шарах σ1 и σ2 после их соединения. Расстояние между шарами велико по сравнению с их радиусами. Емкостью проводника, соединяющего шары, пренебречь.

Решение:
Заряды на шарах до и после соединения Общий потенциал шаров после соединения определим из условия сохранения заряда
Заряды на первом и втором шарах после соединения

Поверхностные плотности зарядов на шарах


21 Плоский воздушный конденсатор, заряженный до разности потенциалов V0 = 800 В, соединили параллельно с таким же по размерам незаряженным конденсатором, заполненным диэлектриком. Какова диэлектрическая проницаемость e диэлектрика, если после соединения разность потенциалов между пластинами конденсаторов оказалась равной V=100В?

Решение:

22 Найти емкость С трех плоских воздушных конденсаторов, соединенных параллельно. Размеры конденсаторов одинаковы: площадь пластины S=314 см2, расстояние между пластинами d=1 мм. Как изменится емкость трех конденсаторов, если пространство между пластинами одного конденсатора заполнить слюдой (диэлектрическая проницаемость ε1 = 7), а другого — парафином (диэлектрическая проницаемость ε2 = 2)?

Решение:
Емкость трех конденсаторов без диэлектрика При заполнении двух конденсаторов диэлектриками емкость трех конденсаторов

23 В заряженном плоском конденсаторе, отсоединенном от источника тока, напряженность электрического поля равна Е0. Половину пространства между пластинами конденсатора заполнили диэлектриком с диэлектрической проницаемостью ε (толщина диэлектрика равна расстоянию между пластинами). Найти напряженность электрического поля Е в пространстве между пластинами, свободном от диэлектрика.

Решение:
Если d-расстояние между пластинами и С0 — емкость конденсатора без диэлектрика, то разность потенциалов между пластинами конденсатора (без диэлектрика) и заряд на пластинах Конденсатор, половина которого заполнена диэлектриком, можно рассматривать как два соединенных параллельно конденсатора (рис. 341), причем один не содержит диэлектрика и имеет емкость а в другом все пространство между пластинами заполнено диэлектриком, и поэтому его емкость Полная емкость конденсатора, половина которого заполнена диэлектриком, При отключенном источнике тока заряд на пластинах сохраняется, поэтому разность потенциалов между пластинами V=q/C, и напряженность электрического поля в пространстве между пластинами, свободном от диэлектрика,


24 Два последовательно соединенных конденсатора с емкостями C1 = 1 мкФ и С2 = 3 мкФ подключены к источнику тока с напряжением V =220 В. Найти напряжение на каждом конденсаторе.

Решение:
Если V1 и V2 — напряжения на первом и втором конденсаторах, то V= V1 + V2, а заряды на них одинаковы и равны
q=C1V1=C2V2; отсюда

При последовательном соединении конденсаторов на конденсаторе меньшей емкости напряжение больше, чем на конденсаторе большей емкости.

25 Два последовательно соединенных конденсатора с емкостями C1 = 1 мкФ и С2 = 2 мкФ подключены к источнику тока с напряжением V =900 В. Возможна ли работа такой схемы, если напряжение пробоя конденсаторов Vпр = 500 В?

Решение:
Напряжения на первом и втором конденсаторах (см. задачу 24). Работать при указанном в условии задачи напряжении пробоя конденсаторов нельзя, ибо произойдет пробой первого, а затем и второго конденсаторов.

26 Два последовательно соединенных конденсатора подключены к источнику тока с напряжением V= 200 В (рис. 79). Один конденсатор имеет постоянную емкость C1 = 0,5 мкФ, а другой — переменную емкость С2 (от Cmin = 0,05 мкФ до Сmах = 0,5 мкФ). В каких пределах изменяется напряжение на переменном конденсаторе при изменении его емкости от минимальной до максимальной?

Решение:
При изменении емкости переменного конденсатора С2 от Cmin до Сmax, напряжение на нем V изменяется в пределах (см. задачу 24)

27 При последовательном соединении трех различных конденсаторов емкость цепи С0 = 1 мкФ, а при параллельном соединении емкость цепи С=11мкФ. Найти емкости конденсаторов С2 и С3, если емкость конденсатора С1 = 2 мкФ.

Решение:

28 При последовательном соединении трех различных конденсаторов емкость цепи С0 = 0,75 мкФ, а при параллельном соединении емкость цепи С = 7 мкФ. Найти емкости конденсаторов С2 и С3 и напряжения на них V2 и V3 (при последовательном соединении), если емкость конденсатора C1 = 3 мкФ, а напряжение на нем V1 = 20B.

Решение:
При последовательном соединении конденсаторов имеем

при параллельном

Из этих уравнений находим

Согласно теореме Виета С2 и С3 должны быть корнями квадратного уравнения

Решая его, найдем

Заряды на всех конденсаторах при последовательном соединении равны между собой:


29 Три последовательно соединенных конденсатора с емкостями С1 = 100пФ, С2 = 200 пФ, С3 = 500 пФ подключены к источнику тока, который сообщил им заряд q=10нКл. Найти напряжения на конденсаторах V1, V2 и V3, напряжение источника тока V и емкость всех конденсаторов С0.

Решение:
При последовательном соединении конденсаторов заряд каждого конденсатора равен q, поэтому

Напряжение источника тока равно полному напряжению на всех конденсаторах:

Так как при последовательном соединении
то

30 Три последовательно соединенных конденсатора с емкостями С1=0,1мкФ, С2 = 0,25 мкФ и С3 = 0,5 мкФ подключены к источнику тока с напряжением V =32 В. Найти напряжения V1, V2 и V3 на конденсаторах.

Решение:


31 Два одинаковых воздушных конденсатора емкости С=100пФ соединены последовательно и подключены к источнику тока с напряжением V= 10 В. Как изменится заряд на конденсаторах, если один из них погрузить в диэлектрик с диэлектрической проницаемостью ε = 2?

Решение:
При последовательном соединении конденсаторов заряды на конденсаторах равны. До погружения одного из них в диэлектрик заряд на каждом конденсаторе

после погружения одного из них в диэлектрик заряды конденсаторов будут

Учитывая, что

Изменение заряда на конденсаторах


32 Два плоских воздушных конденсатора с одинаковыми емкостями соединены последовательно и подключены к источнику тока. Пространство между пластинами одного из конденсаторов заполняют диэлектриком с диэлектрической проницаемостью ε = 9. Во сколько раз изменится напряженность электрического поля Е в этом конденсаторе?

Решение:
Первоначальная напряженность электрического поля в каждом конденсаторе

где d-расстояние между пластинами конденсатора. После заполнения одного конденсатора диэлектриком напряженность электрического поля в нем

Отношение напряженностей


33 Решить предыдущую задачу для случая, когда конденсаторы после зарядки отключаются от источника тока.

Решение:
После отключения конденсатора от источника тока и заполнения его диэлектриком заряд на нем не изменяется:

Напряженность электрического поля в конденсаторе, заполненном диэлектриком,

Отношение напряженностей

34 Два плоских воздушных конденсатора с одинаковыми емкостями С=10пФ соединены последовательно. Насколько изменится емкость конденсаторов, если пространство между пластинами одного из них заполнить диэлектриком с диэлектрической проницаемостью ε = 2?

Решение:
Изменение емкости соединенных конденсаторов


35 В плоский воздушный конденсатор с площадью обкладок S и расстоянием между ними d введена параллельно обкладкам проводящая пластинка, размеры которой равны размерам обкладок, а ее толщина намного меньше d. Найти емкость конденсатора с проводящей пластинкой, если пластинка расположена на расстоянии l от одной из обкладок конденсатора.

Решение:
После введения пластинки образовалось два последовательно включенных конденсатора с емкостями

(рис. 342). Их общую емкость определим из соотношения

где С-первоначальная емкость конденсатора. Таким образом, после введения пластинки при любом ее положении С0 = С.

36 В плоский воздушный конденсатор с площадью обкладок S и расстоянием между ними d введена параллельно обкладкам проводящая пластинка, размеры которой равны размерам обкладок, а толщина dп = d/3

Решение:
Введение проводящей пластинки между обкладками конденсатора приводит к образованию двух последовательно включенных конденсаторов с расстояниями между обкладками d1 и d2 и емкостями

(рис.343). Их общую емкость находим из соотношения

При -первоначальная емкость конденсатора.

37 Плоский воздушный конденсатор заряжен до разности потенциалов V0 = 50 В и отключен от источника тока. После этого в конденсатор параллельно обкладкам вносится проводящая пластинка толщины dп= 1 мм. Расстояние между обкладками d=5 мм, площади обкладок и пластинки одинаковы. Найти разность потенциалов V между обкладками конденсатора с проводящей пластинкой.

Решение:
Емкости конденсатора до и после внесения проводящей пластинки толщины dп (см. задачу 36)
Заряд конденсатора, отключенного от источника тока, не изменяется:

отсюда разность потенциалов между обкладками конденсатора после внесения проводящей пластинки

38 В плоский воздушный конденсатор с площадью обкладок S и расстоянием между ними d вводится параллельно обкладкам диэлектрическая пластинка толщины d1<d/ Диэлектрическая проницаемость пластинки равна ε, площади обкладок и пластинки одинаковы и равны S. Найти емкость конденсатора с диэлектрической пластинкой.

Решение:

Электроемкость. Конденсаторы

Что такое электроемкость проводников

Если у нас есть два проводника, изолированных друг от друга, которым мы сообщаем некоторые заряды (обозначим их соответственно q1 и q2), то между ними возникнет определенная разность потенциалов. Ее величина будет зависеть от формы проводников, а также от исходных величин зарядов. Обозначим такую разность Δφ. Если мы говорим о разности, возникающей в электрическом поле между двумя точками, то ее обычно обозначают U.

В рамках темы данной статьи нам больше всего интересна такая разность потенциалов между проводниками, когда их заряды противоположны по знаку, но равны друг другу по модулю. В таком случае мы можем ввести новое понятие – электрическая емкость (электроемкость).

Определение 1

Электрической емкостью системы, состоящей из двух проводников, называется отношение заряда одного проводника (q) к разности потенциалов между этими двумя проводниками.

В виде формулы это записывается так: C=q∆φ=qU.

Для измерения электрической емкости применяется единица, называемая фарад. Она обозначается буквой Ф.

1Φ=1 Кл1 В.

Конфигурации и размеры проводников, а также свойства диэлектрика определяют величину электроемкости заданной системы. Наибольший интерес для нас представляют проводники особой формы, называемые конденсаторами.

Определение 2

Конденсатор – это проводник, конфигурация которого позволяет локализовать (сосредотачивать) электрическое поле в одной выделенной части пространства. Проводники, составляющие конденсатор, называются обкладками.

Определение 3

Если мы возьмем две плоские пластины из проводящего материала, расположим их на небольшом расстоянии друг от друга и проложим между ними слой диэлектрика, то мы получим простейший конденсатор, называемый плоским. При его работе электрическое поле будет располагаться преимущественно в промежутке между пластинами, но небольшая часть этого поля будет рассеиваться вокруг них.

Определение 4

Часть электрического поля вблизи конденсатора называется полем рассеяния.

Иногда в задачах мы можем не учитывать его и работать только с той частью электрического поля, которое расположено между обкладками. Однако пренебрегать полем рассеяния допустимо далеко не всегда, поскольку это может привести к ошибочным расчетам из-за нарушения потенциального характера электрического поля.

Рисунок 1.6.1. Электрическое поле в плоском конденсаторе.

Рисунок 1.6.2. Электрическое поле конденсатора без учета поля рассеяния, не обладающее потенциальностью.

Модуль напряженности электрического поля, которое создает каждая обкладка в плоском конденсаторе, выражается соотношением следующего вида:

E1=σ2ε0.

Исходя из принципа суперпозиции, можно утверждать, что напряженность E→ поля, которое создают обе пластины конденсатора, будет равна сумме напряженностей E+→ и E-→ полей каждой пластины, то есть E→=E+→+E-→.

Векторы напряженностей обеих пластин во внутренней части конденсатора будут параллельны друг другу. Значит, мы можем выразить модуль напряженности их суммарного поля в виде формулы E=2E1=σε0.

Нужна помощь преподавателя?

Опиши задание — и наши эксперты тебе помогут!

Описать задание

Как рассчитать электроемкость конденсатора

Вне пластин векторы напряженности будут направлены в противоположные друг от друга стороны, значит, E будет равно нулю. Если мы обозначим заряд каждой обкладки как q, а ее площадь как S, то соотношение qS даст нам представление о поверхностной плотности. Умножив E на расстояние между обкладками (d), мы получим разность потенциалов между пластинами в однородном электрическом поле. Теперь возьмем оба этих соотношения и выведем из них формулу, по которой может быть рассчитана электрическая емкость конденсатора.

C=q∆φ=σ·SE·d=ε0Sd.

Определение 5

Электрическая емкость плоского конденсатора – величина, обратно пропорциональная расстоянию между обкладками и прямо пропорциональная их площади.

Заполнение пространства между проводниками диэлектрическим материалом может увеличить электроемкость плоского конденсатора в число раз, кратное undefined.

Определение 6

Введем обозначение емкости в виде буквы С и запишем это в виде формулы:

C=εε0Sd.

Данная формула называется формулой электроемкости плоского конденсатора.

Конденсаторы бывают не только плоскими. Возможны и другие конфигурации, также обладающие специфическими свойствами.

Определение 7

Сферическим конденсатором называется система из 2-х концентрических сфер, сделанных из проводящего материала, радиусы которых равны R1 и R2 соответственно.

Определение 8

Цилиндрическим конденсатором называется системы из двух проводников цилиндрической формы, длина которых равна L, а радиусы R1 и R2.

Обозначим проницаемость диэлектрического материала как ε и запишем формулы, по которым можно найти электрическую емкость конденсаторов:

  • C=4πε0εR1R2R2-R1(сферический конденсатор),
  • C=2πε0εLlnR2R1(цилиндрический конденсатор).

Как рассчитать электроемкость батареи конденсаторов

Определение 9

Если мы соединим несколько проводников между собой, то мы получим конструкцию, называемую батареей.

Способы соединения могут быть разными. Если соединение будет параллельным, то напряжение всех конденсаторов в системе будет одинаково: U1=U2 =U, а заряды можно найти по формулам q1=С1U и q2=C2U. При таком соединении вся система может считаться одним конденсатором, электроемкость которого равна C, заряд – q=q1+q2, а напряжение – U. В виде формулы это выглядит так:

С=q1+q2U или C=C1+C2

Определение 10

Если в батарее конденсаторов элементы соединены параллельно, то для нахождения общей электроемкости нам нужно сложить емкости ее отдельных элементов.

Рисунок 1.6.3. Конденсаторы, соединенные параллельно. C=C1+C2

Рисунок 1.6.4. Конденсаторы, соединенные последовательно: 1C=1C1+1C2

Если же батарея состоит из двух последовательно соединенных конденсаторов, то заряды обоих будут одинаковы: q1=q2=q. Найти их напряжения можно так: U1=qC1 и U2=qC2. Такую систему тоже можно считать одним конденсатором, заряд которого равен q, а напряжение U=U1+U2.

C=qU1+U2 или 1C=1C1+1C2

Определение 11

Если конденсаторы в батарее соединены последовательно, то для нахождения общей электроемкости нам нужно сложить величины, обратные емкостям каждого из них.

Справедливость обеих формул, приведенных выше, не зависит от количества конденсаторов в батарее.

Рисунок 1.6.5. Смоделированное электрическое поле плоского конденсатора.

Емкость плоского и других конденсаторов

Конфигурация конденсатора такова, что поле, которое создается зарядами, локализовано между обкладками. В общем случае электроемкость конденсатора равна:

\[C=\frac{q}{{\varphi }_1-{\varphi }_2}=\frac{q}{U}\left(1\right),\]

где ${\varphi }_1-{\varphi }_2=U$ — разность потенциалов обкладок, которую называют напряжением и обозначают $U$. Емкость по определению считается положительной величиной. Она зависит только от геометрии обкладок конденсатора их взаиморасположения и диэлектрика. Форму обкладок и их расположение подбирают так, чтобы внешние поля минимально влияли на внутреннее поле конденсатора. Силовые линии поля конденсатора начинались на проводнике с положительным зарядом и заканчивались на проводнике с отрицательным зарядом. Конденсатор может быть проводником, который помещен в полость, окруженную замкнутой оболочкой.

В соответствии с конфигураций конденсаторов можно выделить три большие группы: плоские, сферические и цилиндрические (по форме обкладок). Вычисление емкости конденсатора сводится к определению $напряжения$ конденсатора при известном заряде на его обкладках.

Плоский конденсатор

Плоский конденсатор (рис.1) — это две разноименно заряженные пластины, разделенные тонким слоем диэлектрика. Формула для расчета емкости такого конденсатора представляет собой выражение:

\[С=\frac{\varepsilon {\varepsilon }_0S}{d}\left(2\right),\]

где $S$ — площадь обкладки, $d$ — расстояние между обкладками, $\varepsilon $ — диэлектрическая проницаемость вещества. Чем меньше $d$, тем больше совпадает расчётная емкость конденсатора (2), с реальной емкостью.

Рис. 1

Электроемкость плоского конденсатора, заполненного N слоями диэлектрика, толщина слоя с номером i равна $d_i$, диэлектрическая проницаемость этого слоя ${\varepsilon }_i$ вычисляется по формуле:

\[C=\frac{{\varepsilon }_0S}{\frac{d_1}{{\varepsilon }_1}+\frac{d_2}{{\varepsilon }_2}+\dots +\frac{d_N}{{\varepsilon }_N}}\ \left(3\right).\]

Сферический конденсатор

В том случае, если внутренний проводник шар или сфера, внешняя замкнутая оболочка — концентрическая ему сфера, то конденсатор является сферическим. Сферический конденсатор (рис.2) состоит из двух концентрических проводящих сферических поверхностей с пространством между обкладками, заполненным диэлектриком. Емкость его можно рассчитать по формуле:

\[C=4\pi \varepsilon {\varepsilon }_0\frac{R_1R_2}{R_2-R_1}\ \left(4\right),\]

где $R_1{\ и\ R}_2$ — радиусы обкладок.

Рис. 2

Цилиндрический конденсатор

Емкость цилиндрического конденсатора равна:

\[C=\frac{2\pi \varepsilon {\varepsilon }_0l}{{ln \left({R_2}/{R_1}\right)\ }}\left(5\right),\]

где $l$ — высота цилиндров, $R_1$ и $R_2$ — радиусы обкладок. Этот вид конденсаторов представляет собой две коаксиальных (соосных) проводящих цилиндрических поверхности (рис.3).

Рис. 3

Еще одной, но не маловажной характеристикой всех конденсаторов является пробивное напряжение ($U_{max}$)— это напряжение, при котором происходит электрический разряд через слой диэлектрика. $U_{max}$ зависит от толщины слоя и свойств диэлектрика, конфигурации конденсатора.

Помимо одиночных конденсаторов применяют их соединения. Для того чтобы увеличить емкость используют параллельное соединение конденсаторов (соединение одноименными обкладками). В этом случае результирующая емкость такого соединения может быть найдена как сумма${\ С}_i$ где $С_i$ — емкость конденсатора с номером i:

\[C=\sum\limits^N_{i=1}{С_i}\ \left(6\right).4\frac{В}{м}.$

На рисунке представлена схема смешанного соединения конденсаторов

Автор На чтение 10 мин. Опубликовано

При параллельном соединении конденсаторов к каждому кон­денсатору приложено одинаковое напряжениеU, а величина за­ряда на обкладках каждого конденсатора Q пропорциональна его емкости (рис. 2).

Общий заряд Q всех конденсаторов

Общая емкость С, или емкость батареи, параллельно включенных конденсаторов равна сумме емкостей этих конденсаторов.

Параллельное подключение конденсатора к группе других включенных конденсаторов увеличивает общую емкость батареи этих конденсаторов. Следовательно, параллельное соединение конденсаторов при­меняется для увеличения емкости.

4)Если параллельно включены т одинаковых конденсаторов ем­костью С´ каждый, то общая (эквивалентная) емкость батареи этих конденсаторов может быть определена выражением

Последовательное соединение конденсаторов

На обкладках последовательно соединенных конденсаторов, подключенных к источнику постоянного тока с напряжением U, появятся заряды одинаковые по величине с противоположными знаками.

Напряжение на конденсаторах распределяется обратно пропорционально емкостям конденса­торов:

Обратная величина общей емкости последовательно соединенных конденсаторов равна сумме обратных величин емкостей этих кон­денсаторов.

При последовательном включении двух конденсаторов их об­щая емкость определяется следующим выражением:

Если в цепь включены последовательно п одинаковых конден­саторов емкостью С каждый, то общая емкость этих конденса­торов:

Из (14) видно, что, чем больше конденсаторов п соединено последовательно, тем меньше будет их общая емкость С, т. е. по­следовательное включение конденсаторов приводит к уменьше­нию общей емкости батареи конденсаторов.

На практике может оказаться , что допустимое ра­бочее напряжение Up конденсатора меньше напряжения, на кото­рое необходимо подключить конденсатор. Если этот конденсатор подключить на такое напряжение, то он выйдет из строя, так как будет пробит диэлектрик. Если же последовательно включить не­сколько конденсаторов, то напряжение распределится между ними и на каждом конденсаторе напряжение окажется мень­ше его допустимого рабочего Up. Следовательно, последовательное соединение конденсаторов применяют для того, чтобы напряжение на каждом конденсаторе не превышало его рабочего напряжения Up.

Смешанное соединение конденсаторов

Смешанное соединение (последовательно-параллельное) кон­денсаторов применяют тогда, когда необходимо увеличить ем­кость и рабочее напряжение батареи конденсаторов.

Рассмотрим смешанное соединение конденсаторов на ниже­приведенных примерах.

где Q — заряд конденсатора или конденсаторов, к которым при­ложено напряжение U; С — электрическая емкость конденсатора или батареи соединенных конденсаторов, к которой приложено напряжение U.

Таким образом, конденсаторы служат для накопления и сохра­нения электрического поля и его энергии.

15.Дайте определение понятиям трех лучевая звезда и треугольник сопротивлений. Запишите формулы для преобразования трех лучевой звезды сопротивлений в треугольник сопротивлений и наоборот. Преобразуйте схему к двум узлам (Рисунок 5)

Рисунок 5- Схема электрическая

Для облегчения расчета составляется схема замещения электрической цепи, т. е. схема, отображающая свойства цепи при определенных условиях.

На схеме замещения изображают все элементы, влиянием которых на результат расчета нельзя пренебречь, и указывают также электрические соединения между ними, которые имеются в цепи.

1.Схемы замещения элементов электрических цепей

На расчетных схемах источник энергии можно представить ЭДС без внутреннего сопротивления, если это сопротивление мало по сравнению с сопротивлением приемника (рис. 3.13,6).

Приr= 0 внутреннее падение напряженияUо = 0, поэтому

напряжение на зажимах источника при любом токе равно

В некоторых случаях источник электрической энергии на расчетной схеме заменяют другой (эквивалентной) схемой (рис. 3.14, а), где вместо ЭДСЕ источник характеризуется его током короткого замыканияIK, а вместо внутреннего со­противления в расчет вводится внутренняя проводимостьg=1/r.

Возможность такой замены можно доказать, разделив равенство (3.1) на r:

где U/r = Io—некоторый ток, равный отношению напряжения на зажимах источника к внутреннему сопротивлению;E/r = IK — ток короткого замыкания источника;

Вводя новые обозначения, получим равенство IK= Io + I, которому удовлетворяет эквивалентная схема рис. 3.14,а.

В этом случае при любой величине напряжения на зажимах; источника его ток остается равным току короткого замыкания (рис. 3.14,6):

Источник с неизменным током, не зависящим от внешнего сопротивления, называют источником тока.

Один и тот же источник электрической энергии может быть заменен в расчетной схеме источником ЭДС или источником тока.

Смешанным соединением конденсаторов называется такое соединение их, при котором имеется и параллельное и последовательное соединение

При смешанном соединении конденсаторов для участков с параллельным соединением применяются свойства параллельного соединения конденсаторов, а для участков с последовательным соединением – все свойства последовательного соединения конденсаторов.

Всякое смешанное соединение конденсаторов путем упрощений может быть сведено либо к параллельному соединению, либо к последовательному.

Эквивалентная емкость верхней ветви

Эквивалентная емкость нижней цепи

Теперь это смешанное соединение конденсаторов может быть приведено к параллельному соединению. Эквивалентная емкость всей батареи конденсаторов

Эквивалентная емкость между точками 1 и 2:

Эквивалентная емкость между точками 2 и 3

Теперь это смешанное соединение конденсаторов может быть приведено к последовательному соединению

Эквивалентная емкость батареи конденсаторов

Конденсатор емкостью С=2 мкф и номинальным рабочим напряжением Up=600 в вышел из строя.

Составить схему замены его конденсаторами емкостью С=1 мкф и номинальным рабочим напряжением Up=200 в

Р е ш е н и е .Конденсаторы с номинальным рабочим напряжением 200 в нельзя включать под напряжение 600в. Поэтому прежде всего необходимо обеспечить электрическую прочность батареи. Для этого конденсаторы надо соединить последовательно. Число последовательно соединенных конденсаторов должно быть

Емкость такой ветви

Для обеспечения емкости батареи необходимо соединить несколько параллельных ветвей. Число параллельных ветвей

Преподаватель междисциплинарного курса

Практическое занятие №4

Тема: Расчет общей емкости конденсатора

Цель: рассчитать напряжение, заряд, емкость конденсаторов и их энергию в электрических цепях постоянного тока с последовательным, параллельным и смешанным соединением конденсаторов.

1. Изучить краткие теоретические сведения.

2. Рассмотреть примеры выполнения задания.

3. Выполнить индивидуальное задание.

1. Краткие теоретические сведения

Конденсатор – электронный компонент, предназначенный для накопления электрического заряда. Способность конденсатора накапливать электрический заряд зависит от его главной характеристики – емкости. Емкость конденсатора (С) определяется как соотношение количества электрического заряда (Q) к напряжению (U).

где С емкость конденсатора, Ф

Q – заряд конденсатора, Кл

U – напряжение на конденсаторе, В

Энергия конденсатора зависит от его емкости. Поэтому при изменении емкости заряженного конденсатора будем изменяться его энергия:

где W – энергия конденсатора, Дж

В электрических цепях применяются различные способы соединения конденсаторов.

Соединение конденсаторов может производиться: последовательно, параллельно и смешанно (то есть последовательно-параллельно). Существующие виды соединения конденсаторов показаны на рисунке 1.

Рисунок 1. Способы соединения конденсаторов.

Параллельное соединение конденсаторов.

Если группа конденсаторов включена в цепь таким обра­зом, что к точкам включения непосредственно присоединены пластины всех конденсаторов, то такое соединение называется параллельным соединением конденсаторов (рисунок 2.).

Рисунок 2. Параллельное соединение конденсаторов.

При заряде группы конденсаторов, соединенных параллель­но, между пластинами всех конденсаторов будет одна и та же разность потенциалов, так как все они заряжаются от одного и того же источника тока.

U = UC1 = UC2 = UC3 = …

Общее же количе­ство электричества на всех конденсаторах будет равно сумме количеств электричества, помещающихся на каждом из кон­денсаторов, так как заряд каждого их конденсаторов проис­ходит независимо от заряда других конденсаторов данной группы:

Исходя из этого, всю систему параллельно соединен­ных конденсаторов можно рассматривать как один эквива­лентный (равноценный) конденсатор. Тогда общая емкость конденсаторов при параллельном соединении равна сумме емкостей всех соединенных конденсаторов.

Обозначим суммарную емкость соединенных в батарею конденсаторов бук­вой Собщ, емкость первого конденсатора С1 емкость второго С2 и емкость третьего С3. Тогда для параллельного соединения конденсаторов будет справедлива следующая формула:

Собщ = С1 + С2 + С3 + …

Последний знак + и многоточие указывают на то, что этой формулой можно пользоваться при четырех, пяти и во­обще при любом числе конденсаторов.

Последовательное соединение конденсаторов.

Если же соединение конденсаторов в батарею производится в виде цепочки и к точкам включения в цепь непосредственно присоединены пластины только первого и последнего конденсаторов, то такое соединение конденсаторов называется последо­вательным (рисунок 3).

Рисунок 3. Последовательное соединение конденсаторов.

При последовательном соединении все конденса­торы заряжаются одинаковым количеством электричества, так как непосредственно от источника тока заряжаются только крайние пластины (1 и 6), а остальные пластины (2, 3, 4 и 5) заря­жаются через влияние. При этом заряд пла­стины 2 будет равен по величине и противо­положен по знаку за­ряду пластины 1, заряд пластины 3 будет равен по величине и противоположен по знаку заряду пла­стины 2 и т. д. Всю группу конденсаторов, соединенных последовательно, можно рассмотреть как один эквивалентный конденсатор, между пластинами которого существует напряжение, равное сумме напряжений на всех конденсаторах группы, а заряд которого равен заряду любого из конденсаторов группы.

Напряжения на различных конденсаторах будут различными, так как для заряда одним и тем же количеством электричества конденсаторов различной емкости всегда требуются различные напряжения.

U = UC1 + UC2 + UC3 + …

Чем меньше емкость конденсатора, тем большее напряжение необходимо для того, чтобы зарядить этот конденсатор требуемым количеством электричества, и наоборот.

Таким образом, при заряде группы конденсаторов, соединенных последовательно, на конденсаторах малой емкости напряжения будут больше, а на конденсаторах большой емкости — меньше.

Для вычисления общей емкости при последовательном со­единении конденсаторов удобнее всего пользоваться следую­щей формулой:

Для частного случая двух последовательно соединенных конденсаторов рисунок 4

Рисунок 4. Последовательное соединение двух конденсаторов

Формула для вычисления их общей емкости будет иметь вид:

Последовательно-параллельным соединением конденсаторов называется цепь имеющая в своем составе участки, как с параллельным, так и с последовательным соединением конденсаторов.

На рисунке 5 приведен пример участка цепи со смешанным соединением конденсаторов.

Рисунок 5. Последовательно-параллельное соединение конденсаторов.

При расчете общей емкости такого участка цепи с последовательно-параллельным соединением конденсаторов этот участок разбивают на простейшие участки, состоящие только из групп с последовательным или параллельным соединением конденсаторов.

Всякое смешанное соединение конденсаторов путем упрощений может быть сведено либо к параллельному соединению, либо к последовательному.

Алгоритм расчета имеет вид:

1. Определяют эквивалентную емкость участков с последовательным соединением конденсаторов.

2. Если эти участки содержат последовательно соединенные конденсаторы, то сначала вычисляют их емкость.

3. После расчета эквивалентных емкостей конденсаторов перерисовывают схему. Обычно получается цепь из последовательно соединенных эквивалентных конденсаторов.

4. Рассчитывают емкость полученной схемы.

Один из примеров расчета емкости при смешанном соединении конденсаторов приведен на рисунке 6.

Рисунок 6. Алгоритм сворачивания схемы при смешанном соединении конденсаторов

2. Пример выполнения задания

Дано: Решение:

U = 36 В 1. Конденсаторы С1 и С2 соединены параллельно:

Собщ, q1, q2, q3, С1,2 = С1 + С2 = 8×10-6 + 4×10-6 = 12×10-6 Ф = 12 мкФ

q4, q, W1, W2, 2. Конденсаторы С3 и С4 соединены последовательно:

W3, W4, W – ?

3. Общая емкость:

Собщ =

4. Определим заряд цепи:

q = Собщ×U = 2×10-6×36 = 72×10-6 Кл = 72 мкКл

При последовательном соединении конденсаторов:

q = q1,2 = q3 = q4 = 72 мкКл

Тогда, напряжение на участках цепи

U1,2 =

U3 =

U4 =

U1,2 = U1 = U2 = 6 В, тогда

q1 = C1×U1 = 8×10-6×6 = 48×10-6 Кл = 48 мкКл

q2 = C2×U2 = 4×10-6×6 = 24×10-6 Кл = 24 мкКл

5. Энергия всей цепи:

W = = 1,29 мДж

Энергия электрического поля каждого конденсатора:

W1 =

W2 =

W3 =

W4 =

Ответ: Собщ = , q1 = 48 мкКл, q2 = 24 мкКл, q = q3 = q4 = 72 мкКл,

W1 , W2 , W3 , W4 ,

Эквивалентная емкость верхней ветви

Эквивалентная емкость нижней цепи

Теперь это смешанное соединение конденсаторов может быть приведено к параллельному соединению. Эквивалентная емкость всей батареи конденсаторов

Эквивалентная емкость между точками 1 и 2:

Эквивалентная емкость между точками 2 и 3

Теперь это смешанное соединение конденсаторов может быть приведено к последовательному соединению

Эквивалентная емкость батареи конденсаторов

Конденсатор емкостью С=2 мкф и номинальным рабочим напряжением Up=600 в вышел из строя.

Составить схему замены его конденсаторами емкостью С=1 мкФ и номинальным рабочим напряжением Up=200 В.

Р е ш е н и е. Конденсаторы с номинальным рабочим напряжением 200 В нельзя включать под напряжение 600 В. Поэтому прежде всего необходимо обеспечить электрическую прочность батареи. Для этого конденсаторы надо соединить последовательно. Число последовательно соединенных конденсаторов должно быть

Емкость такой ветви

Для обеспечения емкости батареи необходимо соединить несколько параллельных ветвей. Число параллельных ветвей

Общая схема замены конденсатора

3. Индивидуальные задания для обучающихся

Задание: Определить эквивалентную емкость цепи, напряжение на каждом конденсаторе, заряд и энергию электрического поля в цепи и для каждого конденсатора.

Варианты задания, номера схемы и исходные данные для расчета приведены в таблице 1.

Lab 2 — Конденсаторы

Введение

Основная функция конденсатора — накапливать заряд и, следовательно, электрическую энергию. Эта энергия может быть извлечена позже для различных целей. Часто несколько конденсаторов объединяются в единую цепь. Есть два основных способа соединения конденсаторов: последовательно и параллельно. В этой лабораторной работе вы узнаете, как накапливаются заряд и энергия как в последовательной, так и в параллельной комбинациях конденсаторов.

Обсуждение принципов

Конденсатор представляет собой набор из двух металлических проводников, разделенных небольшим расстоянием. Обычно между двумя проводниками помещается какой-либо диэлектрический (непроводящий) материал. Примером может служить конденсатор с параллельными пластинами, показанный на рис.1. При подключении к источнику напряжения, например к батарее, две проводящие пластины заряжаются. При первом подключении батареи свободные электроны внутри верхней пластины конденсатора будут двигаться к положительной клемме батареи.Это продолжается до тех пор, пока верхняя пластина конденсатора не будет иметь тот же потенциал, что и положительный вывод батареи. Чистый положительный заряд + Q будет на верхней пластине конденсатора. В то же время свободные электроны внутри провода, соединяющего отрицательный вывод с нижней пластиной конденсатора, будут двигаться к нижней пластине конденсатора. Это продолжается до тех пор, пока нижняя пластина конденсатора не будет иметь тот же потенциал, что и отрицательная клемма батареи. Чистый отрицательный заряд — Q будет присутствовать на нижней пластине конденсатора.

Рисунок 1 : Цепь конденсатора

На рис. 1 (a) показан конденсатор с параллельными пластинами, подключенный к батарее, а на рис. 1 (b) представлена ​​принципиальная схема той же установки. Обратите внимание, что символы конденсатора и батареи похожи, но не одинаковы. Будьте осторожны, не перепутайте их. Обратите внимание, что на двух пластинах заряженного конденсатора есть равные и противоположные заряды (+ Q и — Q ). Для удобства мы просто ссылаемся на величину заряда и говорим, что на конденсаторе есть заряд Q .Также обратите внимание, что поскольку верхняя пластина конденсатора имеет тот же потенциал, что и положительный вывод батареи, а нижняя пластина имеет тот же потенциал, что и отрицательный вывод батареи, разность потенциалов между двумя пластинами конденсатора такая же, как разность потенциалов

ΔV.

на выводах аккумуляторной батареи. Количество заряда, нанесенного на конденсатор, прямо пропорционально разности потенциалов на его пластинах. Это можно записать как где константа пропорциональности C известна как емкость конденсатора и измеряется в единицах Фарад (Ф).Емкость зависит от конкретной геометрии конденсатора, а также от типа используемого диэлектрического материала. Если вы удалите батарею из заряженного конденсатора, заряды останутся на пластинах, и между двумя пластинами конденсатора останется разность потенциалов. Поэтому мы говорим, что конденсатор «хранит» электрическую энергию. Количество энергии, хранящейся в заряженном конденсаторе, можно рассчитать по формуле Используя уравнение. (1)

Q = CΔV

, эту формулу можно переписать в двух других эквивалентных формах.

Конденсаторы, подключенные параллельно

Часто нам нужно объединить несколько конденсаторов в одну схему. Одним из способов объединения конденсаторов является их параллельное соединение, как показано на рис. 2. Обратите внимание, что верхние пластины обоих конденсаторов подключены к положительной клемме батареи, а нижние пластины обоих конденсаторов подключены к отрицательной клемме. Следовательно, разность потенциалов на каждом отдельном конденсаторе будет такой же, как полная разность потенциалов, размещаемая батареей во всем параллельном устройстве.

(5)

ΔV 1 = ΔV 2 = ΔV

Каждый отдельный конденсатор будет хранить разные заряды, Q 1 и Q 2 . Общий заряд, помещенный аккумулятором на всю параллельную схему, затем

(6)

Q итого = Q 1 + Q 2 .

Рисунок 2 : Эскиз и схема двух конденсаторов, подключенных параллельно

Общая емкость параллельной схемы,

C, , общая ,

, будет отличаться от любой из отдельных емкостей, C 1 или C 2 .Чтобы определить эту эквивалентную емкость, сначала обратите внимание, что мы можем использовать уравнения. (1)

Q = CΔV

и (5)

ΔV 1 = ΔV 2 = ΔV

для записи

Q всего = C экв ΔV, Q 1 = C 1 ΔV, и Q 2 = C 2 ΔV.

Подставляя эти выражения в Ур. (6)

Q итого = Q 1 + Q 2 .

и отбрасывая общий множитель

ΔV

, получаем

(7)

C итого = C 1 + C 2 .

Это обычно называется эквивалентной емкостью и обозначается

C eq .

Итак, для параллельной схемы, состоящей из Н различных конденсаторов,

C экв = C 1 + C 2 + C 3 + … + C N.

Энергия

E всего

, сохраненных в параллельном расположении, можно рассчитать с помощью уравнения. (2) как

(8)

E всего = C экв (ΔV) 2
= (C 1 + C 2 ) (ΔV) 2
= C 1 (ΔV) 2 + C 2 (ΔV) 2 .

Обратите внимание, что общая энергия, запасенная в параллельном расположении, равна сумме энергий, сохраненных в каждом из отдельных конденсаторов. Конденсаторы

, подключенные последовательно

Другой способ объединения конденсаторов — их последовательное соединение, как показано на рис. 3. При подключении к батарее нижняя пластина первого конденсатора C 1 будет накапливать заряд — Q , а верхняя пластина конденсатора второй конденсатор C 2 будет хранить заряд + Q .Таким образом, мы видим, что общий заряд батареи составляет Q . Обратите внимание, что верхняя пластина C 1 и нижняя пластина C 2 соединены проводом, образуя единый H-образный проводник. Этот проводник не подключен к аккумулятору, поэтому аккумулятор не может напрямую обеспечивать его зарядом. Вместо этого отрицательный заряд — Q на нижней пластине C 1 и положительный заряд + Q на верхней пластине C 2 приведет к тому, что свободные электроны в H-образном проводнике будут перейти наверх.Результатом является индуцированный чистый заряд + Q на верхней пластине C 1 и индуцированный чистый заряд — Q на нижней пластине C 2 . Следовательно, оба конденсатора будут иметь одинаковый заряд Q , который совпадает с общим зарядом всей конструкции.

(9)

Q итого = Q 1 = Q 2

Рисунок 3 : Эскиз и схема двух последовательно соединенных конденсаторов

Нижняя пластина C 1 имеет тот же потенциал, что и отрицательный вывод аккумулятора, а верхняя пластина C 2 имеет тот же потенциал, что и положительный вывод.Н-образный проводник будет иметь потенциал где-то между этими двумя значениями. Таким образом, при переходе от верхней пластины к нижней пластине C 2 потенциал упадет на

ΔV 2 .

Затем, переходя от верхней пластины к нижней пластине C 1 , потенциал упадет на

ΔV 1 .

Разность потенциалов

ΔV

по всей последовательной схеме равна сумме разностей потенциалов на каждом отдельном конденсаторе.

(10)

ΔV = ΔV 1 + ΔV 2

Мы можем использовать уравнение. (1)

Q = CΔV

, чтобы записать разность потенциалов на отдельных конденсаторах и параллельном расположении как Подставляя эти выражения в уравнение. (10)

ΔV = ΔV 1 + ΔV 2

, используя уравнение. (9)

Q итого = Q 1 = Q 2

, и отбрасывая общий множитель,

Q итого ,

дает Энергия, запасенная в общей последовательности расположения, может быть рассчитана с помощью уравнения.(4).

(15)

= Q итого ΔV 1 + Q итого ΔV 2
E всего = Q всего ΔV
= Q всего (ΔV 1 + ΔV 2 )

Обратите внимание, что это сумма энергий, хранящихся в каждом отдельном конденсаторе.Ниже перечислены уравнения, управляющие распределением напряжения, распределением заряда, эквивалентной емкостью и накоплением энергии для двух типов устройств.

Параллельное расположение Последовательное расположение
ΔV = ΔV 1 = ΔV 2 ΔV = ΔV 1 ΔV = ΔV 1 + ΔV 2 9015 901 9015 901 9014 900 = Q 1 + Q 2 Q итого = Q 1 = Q 2
C экв = C 1 + C 2 = +
E итого = E 1 + E 2 E итого = E 1 + E 2

Цель

Цель этого эксперимента — изучить характеристики конденсаторов, подключенных последовательно и параллельно, и качественно исследовать энергию, запасаемую в таких схемах.

Оборудование

  • Сигнальный интерфейс с выходной мощностью
  • Программное обеспечение Capstone
  • Мультиметр
  • Измеритель емкости
  • Два разных конденсатора
  • Один светодиод
  • Секундомер

Процедура

Распечатайте лист для этой лабораторной работы.Этот лист понадобится вам для записи ваших данных.

Эта лабораторная работа состоит из четырех частей. Первая процедура — качественное исследование энергии, запасаемой конденсаторами различной конфигурации. Вы будете заряжать конденсаторы как при последовательном, так и при параллельном расположении конденсаторов и в каждом случае использовать накопленную энергию, чтобы зажечь небольшую светодиодную лампочку. Вы сможете предсказать, в каком устройстве хранится больше энергии, исходя из количества времени, в течение которого лампочка остается горящей. В остальных процедурах вы будете использовать измеритель емкости и вольтметр для количественных измерений параллельной и последовательной компоновки конденсаторов.Вы сравните свои данные с теоретическими уравнениями для распределения напряжения, распределения заряда, эквивалентной емкости и накопления энергии.

Контрольная точка:
Перед подключением источника питания к цепи попросите своего технического специалиста проверить соединения вашей цепи, чтобы убедиться, что ваши конденсаторы подключены правильно.

Процедура A: зажигание светодиода

1

Откройте соответствующий файл Capstone, связанный с этой лабораторной работой.На экране должно быть окно генератора сигналов, подобное изображенному на рис. 4. Кабели с банановыми вилками должны быть подключены к выходу 1.

2

В окне генератора сигналов выберите DC. Установите напряжение постоянного тока на 5.0.

Рисунок 4 : Окно генератора сигналов

3

Разрядите оба конденсатора, замкнув их как минимум на 10 секунд, как показано на рис.5. Вы можете проверить, полностью ли разряжены конденсаторы, используя вольтметр, чтобы определить разность потенциалов на каждом конденсаторе.См. Приложение К. Показания должны быть очень близки к 0,0 вольт.

Рисунок 5 : Разряд конденсатора

4

Соедините два конденсатора последовательно. Они должны быть подключены так, чтобы стрелки на каждом конденсаторе были направлены в том же направлении, что и на рис.6. Красный выходной провод должен быть подключен к концу одного конденсатора с выемкой (это также конец с черной лицевой стороной), а черный провод должен быть подключен к концу, на который стрелки указывают на него (это конец с серебряным концом). лицо).Это зарядит конденсаторы.

Рисунок 6 : Принципиальная схема и фото конденсаторов в серии

5

Отключите вывод питания от цепи, отсоединив красный и черный провода от конденсаторов. НЕ ОТКЛЮЧАЙТЕ выходную мощность или обнуляйте ее напряжение.

6

Подключите светодиод к конденсатору, как показано на рис.7. Красный и черный провода светодиодов должны быть подключены так же, как красный и черный провода усилителя мощности.Немедленно начните отсчет времени, как долго светодиод будет гореть. Запишите время на листе.

Рисунок 7 : Принципиальная схема и фото для подключения светодиода

7

Отключите последовательное соединение и полностью разрядите оба конденсатора.

8

Подключите два конденсатора параллельно. Опять же, стрелки должны указывать в том же направлении, что и на рис.8.

Рисунок 8 : Принципиальная схема и фото конденсаторов, подключенных параллельно

9

Повторите шаги с 3 по 6 для параллельного расположения, как показано на рис.8. Запишите время параллельного расположения на листе.

10

Теперь используйте только конденсатор

1000- μ F

и посмотрите, как долго светодиод будет гореть. Запишите это время на листе.

Контрольная точка 1:
Попросите своего технического специалиста проверить вашу работу, прежде чем продолжить.

Процедура B

11

Установите соответствующий параметр на измерителе емкости.Измерьте емкость каждого отдельного конденсатора. Запишите емкости в Таблицу данных 1.

12

Подключите два конденсатора параллельно.

13

С помощью измерителя емкости измерьте общую (или эквивалентную) емкость параллельной схемы. Запишите это значение в Таблицу данных 1.

14

Оставьте параллельную схему подключенной и разрядите конденсаторы, закоротив их по одному.

15

Используйте мультиметр, чтобы убедиться, что они полностью разряжены.См. Приложение К.

16

Подключите параллельную схему к выводам мощности, как показано на рис.8.

17

Включите мультиметр и переключитесь на нужную шкалу постоянного напряжения.

18

На этом этапе важно, чтобы вы не касались вывода конденсатора при измерении напряжения. Вы являетесь резистором, как и светодиод, поэтому вы можете разрядить конденсатор, прикоснувшись к нему. Измерьте и запишите разность потенциалов

ΔV 1 и ΔV 2

на C 1 и C 2 соответственно.Запишите эти значения в Таблицу данных 1.

19

Измерьте разность потенциалов

ΔV p

в параллельном соединении и запишите ее в Таблицу данных 1.

20

Используя измеренные значения емкости и напряжения и уравнения. (1)

Q = CΔV

, рассчитайте заряды для отдельных конденсаторов и параллельной схемы. Запишите эти значения в Таблицу данных 1.

22

Используя уравнение. (7)

C итого = C 1 + C 2 .

, рассчитайте теоретическую эквивалентную емкость для двух параллельно включенных конденсаторов. Запишите это значение на листе.

23

Вычислите процентную разницу между теоретическим значением и измеренным значением (см. Таблицу данных 1) эквивалентной емкости. См. Приложение Б.

24

Используя уравнение. (6)

Q итого = Q 1 + Q 2 .

, рассчитайте теоретический общий заряд для двух конденсаторов, включенных параллельно. Запишите это значение на листе.

25

Вычислите разницу в процентах между теоретическим значением и измеренным значением общего заряда и запишите его в рабочий лист.

Checkpoint 2:
Попросите своего технического специалиста проверить вашу таблицу и расчеты.

Процедура C: Последовательные конденсаторы

26

Запишите ранее измеренные емкости двух конденсаторов в Таблицу данных 2.

27

Соедините два конденсатора последовательно.

28

С помощью измерителя емкости измерьте общую емкость последовательного соединения. Запишите это значение в Таблицу данных 2.

29

Оставьте соединение последовательно и разрядите конденсаторы, закоротив их по одному. Убедитесь, что они полностью разряжены.

30

Подключите усилитель мощности к устройству, как показано на рис.6.

31

Измерьте напряжение на каждом отдельном конденсаторе.Запишите свои измерения в Таблицу данных 2. Опять же, будьте осторожны, чтобы не прикасаться к проводам при записи напряжений.

32

Измерьте напряжение

ΔV s

во всем последовательном соединении. Запишите это значение в Таблицу данных 2.

33

Используя измеренные значения емкости и напряжения, рассчитайте заряд, накопленный на каждом конденсаторе. Запишите эти значения в Таблицу данных 2.

34

Используя измеренные значения для общей емкости и напряжения и уравнения.(1)

Q = CΔV

, рассчитайте заряд при последовательном расположении. Запишите эти значения в Таблицу данных 2.

36

Рассчитайте теоретическую емкость для последовательного соединения, используя формулу. (14). Запишите это значение на листе.

37

Рассчитайте процентную разницу между теоретическим значением и измеренным значением (из таблицы данных 2) эквивалентной емкости. Запишите это на листе.

38

Используя уравнение. (10)

ΔV = ΔV 1 + ΔV 2

, рассчитайте теоретическое напряжение для последовательной схемы.Запишите это значение на листе.

39

Вычислите процентную разницу между этим теоретическим значением и измеренным значением (из Таблицы данных 2) общего напряжения и запишите его в рабочий лист.

Контрольная точка 3:
Попросите своего технического специалиста проверить вашу таблицу и расчеты.

Страница не найдена | MIT

Перейти к содержанию ↓
  • Образование
  • Исследовательская работа
  • Инновации
  • Прием + помощь
  • Студенческая жизнь
  • Новости
  • Выпускников
  • О Массачусетском технологическом институте
  • Подробнее ↓
    • Прием + помощь
    • Студенческая жизнь
    • Новости
    • Выпускников
    • О Массачусетском технологическом институте
Меню ↓ Поиск Меню Ой, похоже, мы не смогли найти то, что вы искали!
Попробуйте поискать что-нибудь еще! Что вы ищете? Увидеть больше результатов

Предложения или отзывы?

Два конденсатора с параллельными пластинами подключены последовательно к батарее, как показано на рис.24-4. А …

  • На рис.1 показана схема из четырех конденсаторов, подключенных к аккумулятор с разностью потенциалов …

    На рис.1 показана схема из четырех конденсаторов, подключенных к аккумулятор с разностью потенциалов? _0. Конденсатор 4 заполнен диэлектрик с 0 диэлектрической проницаемостью? = 4. Емкость номинал конденсаторов равен? 1 =? 2 =? 3 =?. Емкость конденсатора 4 тоже равно? / 2 когда в середине в их пластинах нет диэлектрика. а) Рассчитайте бесплатную заряд, накопленный в конденсаторе 4.Его отвечать…

  • Три конденсатора с параллельными пластинами подключены к батарее, как показано на рисунке. Емкость C составляет 803F …

    Три конденсатора с параллельными пластинами подключены к батарее, как показано на рисунке. Емкость C составляет 803F. Все конденсаторы в цепи имеют одинаковую площадь пластины. Конденсатор Cy наполовину заполнен диэлектрическим материалом (K = 4). Батарея подает 100 В в цепь (рис. 1). Часть A Какой заряд накоплен на C? Выразите свой ответ с помощью трех значащих цифр.лет ΑΣΦ? 2. c Рис. 1 из 1 Отправить запрос Ответ Czi Предоставить отзыв о C₂ 21 Aramak …

  • Заряд конденсаторов последовательно

    Два параллельных пластинчатых конденсатора C1 и C2 соединены последовательно с батареей 50,0 В и батареей 290 k? резистор, как показано на рисунке. Оба конденсатора имеют пластины площадью 1,87 см2 и зазором 0,130 мм. Между пластинами конденсатора C1 воздух, а в конденсаторе C2 зазор заполнен фарфором (диэлектрическая проницаемость 7 и электрическая прочность 5.70 кВ / мм). Переключатель замкнут, и проходит много времени. А) Какой заряд на конденсаторе С1? Б) …

  • Вопрос №11 только пожалуйста. 9. Два конденсатора подключены последовательно с напряжением 12,0 В …

    Вопрос №11 только пожалуйста. 9. Два конденсатора соединены последовательно с батареей 12,0 В. Найдите заряд на каждом конденсаторе. C1 и C2 2,5 ° F 4,5 ° F. C1 10. Два конденсатора подключены параллельно к батарее 6,0 В. Найдите заряд на каждом конденсаторе. С 2.5? F и C, 4,5? F C1 C2 11. Найдите заряд и напряжение на каждом из следующих конденсаторов. Ci-2,00 мкФ. C2-6.00 ??. C3-2.00? F, Ci …

  • Две параллельные пластины, каждая из которых имеет площадь A 3676 см, подключены к клеммам батареи напряжением V, 6 В A, как показано …

    Две параллельные пластины, каждая из которых имеет площадь A 3676 см, подключены к клеммам батареи с напряжением V, 6 В A, как показано. Расстояние между пластинами d 0,42 см. Вы можете предположить (вопреки рисунку), что расстояние между пластинами мало по сравнению с линейным размером пластины + A 1) Что такое C, емкость этого конденсатора с параллельными пластинами? 7.746E-4 мкФ Отправить 2) Что такое Q, заряд хранится на …

  • Два конденсатора C119.0 F и C2 32.0 мкФ подключены последовательно, а батарея на 9,0 В …

    Два конденсатора, C119,0 F и C2 32,0 мкФ, подключены последовательно, и батарея на 9,0 В подключена к ним. (A) Найдите эквивалентную емкость и энергию, содержащуюся в этом эквивалентном конденсаторе. эквивалентная емкость полная запасенная энергия (б) Найдите энергию, запасенную в каждом отдельном конденсаторе. энергия, запасенная в C1 энергия, запасенная в C2. Покажите, что сумма этих двух энергий совпадает с энергией, найденной в части (a).Всегда ли это равенство будет истинным, или …

  • Два конденсатора (C1 = 70 мкФ и C2 = 75 мкФ) подключены в параллельно. А …

    Два конденсатора (C1 = 70 мкФ и C2 = 75 мкФ) подключены в параллельно. Затем последовательно подключают третий конденсатор (C3 = 204 мкФ). с первыми двумя. Затем вся конфигурация помещается в серия с аккумулятором на 8,5 В. Найдите заряд и разность потенциалов на каждом конденсаторе (в мкКл и вольтах, соответственно), как только они будут полностью заряжены.

  • 5. Параллельная комбинация двух идентичных конденсаторов с параллельными пластинами 2,0 мкФ подключена к …

    5. Параллельная комбинация двух идентичных конденсаторов с параллельными пластинами по 2,0 мкФ подключена к батарее на 100 В. Затем батарея удаляется, и расстояние между пластинами одного из конденсаторов увеличивается вдвое. Найдите заряд на каждом конденсаторе 6. Для схемы, показанной ниже, найдите: (a) эквивалентную емкость (b) заряд на каждом конденсаторе (c) напряжение на каждом конденсаторе (d) общую накопленную энергию 0.3 F 10,0 В 1,0 мкФ 0,25 мкФ

  • Конденсаторы C1, C2 и C3 подключен чисто параллельно к аккумулятору на 13 вольт. …

    Конденсаторы C1, C2 и C3 подключен чисто параллельно к аккумулятору на 13 вольт. Заряд в С2 в 2 раза больше заряда в С1, а C3 имеет в 9 раз меньше заряда, чем C2. Общая накопленная энергия во всех трех конденсаторах равна 160 микроджоулей. Какой заряд у C3 в микрокулонах?

  • Разность потенциалов V = 4.200 × 102 В это применяется к двум последовательно включенным конденсаторам. Один конденсатор, …

    Разность потенциалов V = 4.200 × 102В равна применяется к двум последовательно включенным конденсаторам. Один конденсатор, С1, составляет 4,60 ° F, а другой, C2, составляет 7,50 ° F. Заряженные конденсаторы осторожно отключаются от каждого прочее и от АКБ. Затем они снова подключаются, положительный пластина к положительной пластине и отрицательная пластина к отрицательной пластине, с внешнее напряжение не подается. Какая плата за положительная пластина С1? Какова разность потенциалов на C1? Что такое…

  • 4.1 Конденсаторы и емкость — Введение в электричество, магнетизм и схемы

    ЦЕЛИ ОБУЧЕНИЯ

    К концу этого раздела вы сможете:
    • Объясните понятие конденсатора и его емкости
    • Опишите, как оценить емкость системы проводников

    Конденсатор — это устройство, используемое для хранения электрического заряда и электрической энергии. Он состоит как минимум из двух электрических проводников, разделенных расстоянием.(Обратите внимание, что такие электрические проводники иногда называют «электродами», но, точнее, они «обкладки конденсатора».) Пространство между конденсаторами может быть просто вакуумом, и в этом случае конденсатор будет известен как «Вакуумный конденсатор». Однако пространство обычно заполняется изолирующим материалом, известным как диэлектрик . (Вы узнаете больше о диэлектриках в разделах, посвященных диэлектрикам, далее в этой главе.) Объем накопителя в конденсаторе определяется свойством, называемым емкостью , , о котором вы узнаете больше чуть позже в этом разделе.

    Конденсаторы

    имеют различные применения: от фильтрации статического электричества от радиоприема до накопления энергии в дефибрилляторах сердца. Обычно у промышленных конденсаторов две проводящие части расположены близко друг к другу, но не соприкасаются, как на рисунке 4.1.1. В большинстве случаев между двумя пластинами используется диэлектрик. Когда клеммы батареи подключены к первоначально незаряженному конденсатору, потенциал батареи перемещает небольшой заряд величины от положительной пластины к отрицательной пластине.Конденсатор в целом остается нейтральным, но заряжается и находится на противоположных пластинах.

    (рисунок 4.1.1)

    Рисунок 4.1.1. Оба конденсатора, показанные здесь, были изначально разряжены перед подключением к батарее. Теперь у них есть заряды и (соответственно) на своих тарелках. (a) Конденсатор с параллельными пластинами состоит из двух пластин противоположного заряда с площадью A, разделенной расстоянием d. (b) Катаный конденсатор имеет диэлектрический материал между двумя проводящими листами (пластинами).

    Система, состоящая из двух идентичных параллельно проводящих пластин, разделенных расстоянием, называется конденсатором с параллельными пластинами (рисунок 4.1.2). Величина электрического поля в пространстве между параллельными пластинами равна, где обозначает поверхностную плотность заряда на одной пластине (напомним, что это заряд на площадь поверхности). Таким образом, величина поля прямо пропорциональна.

    (рисунок 4.1.2)

    Рисунок 4.1.2. Разделение зарядов в конденсаторе показывает, что заряды остаются на поверхности пластин конденсатора.Линии электрического поля в конденсаторе с параллельными пластинами начинаются с положительных зарядов и заканчиваются отрицательными зарядами. Величина электрического поля в пространстве между пластинами прямо пропорциональна количеству заряда на конденсаторе.

    Конденсаторы с разными физическими характеристиками (такими как форма и размер пластин) накапливают разное количество заряда для одного и того же приложенного напряжения на своих пластинах. Емкость , конденсатора определяется как отношение максимального заряда, который может храниться в конденсаторе, к приложенному напряжению на его пластинах.Другими словами, емкость — это наибольшая величина заряда на вольт, которая может храниться на устройстве:

    (4.1.1)

    Единица измерения емкости в системе СИ — фарад (), названная в честь Майкла Фарадея (1791–1867). Поскольку емкость — это заряд на единицу напряжения, один фарад равен одному кулону на один вольт, или

    По определению, конденсатор способен накапливать заряд (очень большой заряд), когда разность потенциалов между его пластинами равна всего.Следовательно, одна фарада — это очень большая емкость. Типичные значения емкости находятся в диапазоне от пикофарад () до миллифарад (), включая микрофарады (). Конденсаторы могут быть разных форм и размеров (рисунок 4.1.3).

    (рисунок 4.1.3)

    Рисунок 4.1.3 Это некоторые типичные конденсаторы, используемые в электронных устройствах. Размер конденсатора не обязательно зависит от его емкости.

    Расчет емкости

    Мы можем рассчитать емкость пары проводов с помощью следующего стандартного подхода.


    Стратегия решения проблем: расчет емкости

    Чтобы показать, как работает эта процедура, мы теперь вычисляем емкости параллельных пластин, сферических и цилиндрических конденсаторов. Во всех случаях мы предполагаем вакуумные конденсаторы (пустые конденсаторы) без диэлектрического вещества в пространстве между проводниками.

    Конденсатор с параллельными пластинами

    Конденсатор с параллельными пластинами (рисунок 4.1.4) имеет две идентичные проводящие пластины, каждая из которых имеет площадь поверхности, разделенную расстоянием.Когда на конденсатор подается напряжение, он накапливает заряд, как показано на рисунке. Мы можем увидеть, как его емкость может зависеть от и , рассматривая характеристики кулоновской силы. Мы знаем, что сила между зарядами увеличивается с увеличением заряда и уменьшается с расстоянием между ними. Следует ожидать, что чем больше пластины, тем больше заряда они могут хранить. Таким образом, должно быть больше для большего значения. Точно так же, чем ближе пластины друг к другу, тем сильнее на них притяжение противоположных зарядов.Следовательно, должно быть больше за меньшее.

    (рисунок 4.1.4)

    Рисунок 4.1.4. В конденсаторе с параллельными обкладками, разделенными между собой обкладками, каждая обкладка имеет одинаковую площадь поверхности.

    Определим поверхностную плотность заряда на пластинах как

    Из предыдущих глав мы знаем, что когда оно мало, электрическое поле между пластинами довольно однородно (без учета краевых эффектов) и что его величина определяется как

    где постоянная — диэлектрическая проницаемость свободного пространства,.Единица СИ эквивалентна. Поскольку электрическое поле между пластинами однородное, разность потенциалов между пластинами составляет

    Следовательно, уравнение 4.1.3 дает емкость конденсатора с параллельными пластинами как

    (4.1.3)

    Обратите внимание на это уравнение, что емкость является функцией только геометрии и того, какой материал заполняет пространство между пластинами (в данном случае вакуум) этого конденсатора. Фактически, это верно не только для конденсатора с параллельными пластинами, но и для всех конденсаторов: емкость не зависит от или.Если заряд изменяется, соответственно изменяется и потенциал, так что он остается постоянным.

    ПРИМЕР 4.1.1


    Емкость и заряд в конденсаторе с параллельными пластинами

    (a) Какова емкость пустого конденсатора с параллельными пластинами с металлическими пластинами, каждая из которых имеет площадь, разделенную на? (б) Сколько заряда хранится в этом конденсаторе, если к нему приложено напряжение?

    Стратегия

    Определение емкости — это прямое приложение уравнения 4.1.3. Как только мы найдем, мы сможем найти накопленный заряд, используя уравнение 4.1.1.

    Решение

    а. Ввод данных значений в уравнение 4.1.3 дает

    Это небольшое значение емкости указывает на то, насколько сложно сделать устройство с большой емкостью.

    г. Обращение уравнения 4.1.1 и ввод известных значений в это уравнение дает

    Значение

    Этот заряд лишь немного больше, чем в типичных приложениях статического электричества.Поскольку воздух разрушается (становится проводящим) при напряженности электрического поля около, на этом конденсаторе больше не может храниться заряд при увеличении напряжения.

    ПРОВЕРЬТЕ ПОНИМАНИЕ 4.1


    Емкость конденсатора с параллельными пластинами составляет. Если площадь каждой пластины равна, каково расстояние между пластинами?

    ПРОВЕРЬТЕ ПОНИМАНИЕ 4.2


    Убедитесь, что у вас те же физические единицы.

    Сферический конденсатор

    Сферический конденсатор — это еще один набор проводников, емкость которых можно легко определить (Рисунок 4.1.5). Он состоит из двух концентрических проводящих сферических оболочек радиусов (внутренняя оболочка) и (внешняя оболочка). Снарядам придаются равные и противоположные заряды и соответственно. Из-за симметрии электрическое поле между оболочками направлено радиально наружу. Мы можем получить величину поля, применив закон Гаусса к сферической гауссовой поверхности радиусом r , концентричной оболочкам. Вложенная плата есть; поэтому у нас есть

    Таким образом, электрическое поле между проводниками равно

    Мы подставляем это в уравнение 4.1.2 и интегрировать по радиальному пути между оболочками:

    В этом уравнении разность потенциалов между пластинами равна. Подставляем этот результат в уравнение 4.1.1, чтобы найти емкость сферического конденсатора:

    (4.1.4)

    (рисунок 4.1.5)

    Рисунок 4.1.5. Сферический конденсатор состоит из двух концентрических проводящих сфер. Обратите внимание, что заряды на проводнике находятся на его поверхности.

    ПРИМЕР 4.1,3


    Емкость изолированной сферы

    Рассчитайте емкость одиночной изолированной проводящей сферы радиуса и сравните ее с уравнением 4.1.4 в пределе как.

    Стратегия

    Мы предполагаем, что на сфере есть заряд, и поэтому выполняем четыре шага, описанные ранее. Мы также предполагаем, что другой проводник представляет собой концентрическую полую сферу бесконечного радиуса.

    Решение

    На внешней стороне изолированной проводящей сферы электрическое поле задается уравнением 4.1.2. Величина разности потенциалов между поверхностью изолированной сферы и бесконечностью составляет

    Следовательно, емкость изолированного шара составляет

    Значение

    Тот же результат можно получить, взяв предел уравнения 4.1.4 как. Таким образом, одиночная изолированная сфера эквивалентна сферическому конденсатору, внешняя оболочка которого имеет бесконечно большой радиус.

    ПРОВЕРЬТЕ ПОНИМАНИЕ 4.3

    Радиус внешней сферы сферического конденсатора в пять раз превышает радиус его внутренней оболочки.Какие размеры у этого конденсатора, если его емкость?

    Цилиндрический конденсатор

    Цилиндрический конденсатор состоит из двух концентрических проводящих цилиндров (рисунок 4.1.6). Внутренний цилиндр радиуса может быть либо оболочкой, либо полностью твердым. Внешний цилиндр представляет собой оболочку внутреннего радиуса. Мы предполагаем, что длина каждого цилиндра равна и что избыточные заряды и находятся на внутреннем и внешнем цилиндрах соответственно.

    (рисунок 4.1.6)

    Рисунок 4.1.6 Цилиндрический конденсатор состоит из двух концентрических проводящих цилиндров. Здесь заряд на внешней поверхности внутреннего цилиндра положительный (обозначен), а заряд на внутренней поверхности внешнего цилиндра отрицательный (обозначен).

    Без учета краевых эффектов электрическое поле между проводниками направлено радиально наружу от общей оси цилиндров. Используя гауссову поверхность, показанную на рисунке 4.1.6, мы имеем

    Следовательно, электрическое поле между цилиндрами равно

    (4.1,5)

    Здесь \ hat {\ mathrm {r}} — единичный радиальный вектор по радиусу цилиндра. Мы можем подставить в уравнение 4.1.2 и найти разность потенциалов между цилиндрами:

    Таким образом, емкость цилиндрического конденсатора составляет

    (4.1.6)

    Как и в других случаях, эта емкость зависит только от геометрии расположения проводников. Важным применением уравнения 4.1.6 является определение емкости на единицу длины коаксиального кабеля , который обычно используется для передачи изменяющихся во времени электрических сигналов.Коаксиальный кабель состоит из двух концентрических цилиндрических проводников, разделенных изоляционным материалом. (Здесь мы предполагаем наличие вакуума между проводниками, но физика качественно почти такая же, когда пространство между проводниками заполнено диэлектриком.) Эта конфигурация экранирует электрический сигнал, распространяющийся по внутреннему проводнику, от паразитных электрических полей, внешних по отношению к проводнику. кабель. Ток течет в противоположных направлениях во внутреннем и внешнем проводниках, при этом внешний провод обычно заземлен.Теперь из уравнения 4.1.6 емкость коаксиального кабеля на единицу длины равна

    .

    В практических приложениях важно выбирать конкретные значения. Это может быть достигнуто за счет соответствующего выбора радиусов проводников и изоляционного материала между ними.

    ПРОВЕРЬТЕ ПОНИМАНИЕ 4.4


    Когда цилиндрический конденсатор заряжается, между цилиндрами измеряется разность потенциалов.а) Какова емкость этой системы? б) Если цилиндры длинные, каково соотношение их радиусов?

    Несколько типов конденсаторов, пригодных для использования на практике, показаны на рисунке 4.1.3. Обычные конденсаторы часто состоят из двух небольших кусочков металлической фольги, разделенных двумя небольшими кусочками изоляции (см. Рисунок 4.1.1 (b)). Металлическая фольга и изоляция покрыты защитным покрытием, а два металлических вывода используются для подключения фольги к внешней цепи. Некоторые распространенные изоляционные материалы — это слюда, керамика, бумага и антипригарное покрытие Teflon ™.

    Другой популярный тип конденсатора — электролитический конденсатор . Он состоит из окисленного металла в проводящей пасте. Основным преимуществом электролитического конденсатора является его высокая емкость по сравнению с другими распространенными типами конденсаторов. Например, емкость одного типа алюминиевого электролитического конденсатора может достигать. Тем не менее, вы должны быть осторожны при использовании электролитического конденсатора в цепи, потому что он работает правильно только тогда, когда металлическая фольга находится под более высоким потенциалом, чем проводящая паста.Когда возникает обратная поляризация, электролитическое действие разрушает оксидную пленку. Этот тип конденсатора не может быть подключен к источнику переменного тока, потому что в половине случаев переменное напряжение будет иметь неправильную полярность, поскольку переменный ток меняет свою полярность (см. Схемы переменного тока в цепях переменного тока).

    Регулируемый воздушный конденсатор (рисунок 4.1.7) имеет два набора параллельных пластин. Один набор пластин закреплен (обозначен как «статор»), а другой набор пластин прикреплен к валу, который может вращаться (обозначается как «ротор»).Поворачивая вал, можно изменять площадь поперечного сечения в перекрытии пластин; следовательно, емкость этой системы может быть настроена на желаемое значение. Настройка конденсатора находит применение в любом типе радиопередачи и при приеме радиосигналов от электронных устройств. Каждый раз, когда вы настраиваете автомобильное радио на любимую станцию, думайте о емкости.

    (рисунок 4.1.7)

    Рисунок 4.1.7. В конденсаторе переменного тока емкость можно регулировать, изменяя эффективную площадь пластин.(кредит: модификация работы Робби Спрула)

    Обозначения, показанные на рисунке 4.1.8, представляют собой схемные изображения различных типов конденсаторов. Обычно мы используем символ, показанный на рис. 4.1.8 (а). Символ на Рисунке 4.1.8 (c) представляет конденсатор переменной емкости. Обратите внимание на сходство этих символов с симметрией конденсатора с параллельными пластинами. Электролитический конденсатор представлен символом на рис. 4.1.8 (b), где изогнутая пластина обозначает отрицательный вывод.

    (рисунок 4.1.8)

    Рисунок 4.1.8 Здесь показаны три различных схемных представления конденсаторов. Символ в (а) является наиболее часто используемым. Символ в (b) представляет собой электролитический конденсатор. Символ в (c) представляет конденсатор переменной емкости.

    Интересный прикладной пример модели конденсатора взят из клеточной биологии и имеет дело с электрическим потенциалом в плазматической мембране живой клетки (рис. 4.1.9). Клеточные мембраны отделяют клетки от их окружения, но позволяют некоторым отобранным ионам проходить внутрь или из клетки.Разность потенциалов на мембране составляет около. Клеточная мембрана может быть слишком толстой. Рассматривая клеточную мембрану как конденсатор наноразмеров, оценка наименьшей напряженности электрического поля на его «пластинах» дает значение.

    Этой величины электрического поля достаточно, чтобы вызвать электрическую искру в воздухе.

    (рисунок 4.1.9)

    Рисунок 4.1.9. Полупроницаемая мембрана биологической клетки имеет разные концентрации ионов на внутренней поверхности, чем на внешней.Диффузия перемещает ионы (калия) и (хлорида) в показанных направлениях, пока кулоновская сила не остановит дальнейший перенос. Таким образом, внешняя поверхность мембраны приобретает положительный заряд, а ее внутренняя поверхность приобретает отрицательный заряд, создавая разность потенциалов на мембране. Мембрана обычно непроницаема для (ионов натрия).

    Кандела Цитаты

    Лицензионный контент

    CC, конкретная атрибуция

    • Загрузите бесплатно по адресу http: // cnx.org/contents/[email protected]. Получено с : http://cnx.org/contents/[email protected]. Лицензия : CC BY: Attribution

    19.6 Последовательные и параллельные конденсаторы — College Physics

    Сводка

    • Выведите выражения для полной емкости последовательно и параллельно.
    • Обозначение последовательной и параллельной частей в комбинации подключения конденсаторов.
    • Рассчитайте эффективную емкость последовательно и параллельно с учетом индивидуальных емкостей.

    Несколько конденсаторов могут быть соединены вместе в различных приложениях. Несколько подключений конденсаторов действуют как один эквивалентный конденсатор. Общая емкость этого эквивалентного одиночного конденсатора зависит как от отдельных конденсаторов, так и от способа их подключения. Существует два простых и распространенных типа соединений, которые называются серии и параллельно , для которых мы можем легко вычислить общую емкость.Некоторые более сложные соединения также могут быть связаны с комбинациями последовательного и параллельного.

    На рис. 1 (а) показано последовательное соединение трех конденсаторов с приложенным напряжением. Как и для любого конденсатора, емкость комбинации связана с зарядом и напряжением [латексом] {C = \ frac {Q} {V}} [/ latex].

    Обратите внимание на рис. 1, что противоположные заряды величиной [латекс] {Q} [/ латекс] текут в обе стороны от первоначально незаряженной комбинации конденсаторов, когда прикладывается напряжение [латекс] {V} [/ латекс].Для сохранения заряда необходимо, чтобы на пластинах отдельных конденсаторов создавались заряды одинаковой величины, поскольку заряд разделяется только в этих изначально нейтральных устройствах. Конечным результатом является то, что комбинация напоминает одиночный конденсатор с эффективным разделением пластин больше, чем у отдельных конденсаторов. (См. Рисунок 1 (b).) Чем больше расстояние между пластинами, тем меньше емкость. Общей особенностью последовательного соединения конденсаторов является то, что общая емкость меньше любой из отдельных емкостей.

    Рисунок 1. (a) Конденсаторы, подключенные последовательно. Величина заряда на каждой пластине Q . (b) Эквивалентный конденсатор имеет большее расстояние между пластинами d . При последовательном соединении общая емкость меньше, чем у любого из отдельных конденсаторов.

    Мы можем найти выражение для общей емкости, рассматривая напряжение на отдельных конденсаторах, показанных на рисунке 1. Решение [latex] {C = \ frac {Q} {V}} [/ latex] для [latex] {V} [/ latex] дает [latex] {V = \ frac {Q} {C}} [/ latex].Таким образом, напряжения на отдельных конденсаторах составляют [латекс] {V_1 = \ frac {Q} {C_1}} [/ латекс], [латекс] {V_2 = \ frac {Q} {C_2}} [/ латекс] и [ латекс] {V_3 = \ frac {Q} {C_3}} [/ латекс]. Общее напряжение складывается из отдельных напряжений:

    [латекс] {V = V_1 + V_2 + V_3}. [/ Латекс]

    Теперь, называя общую емкость [латекс] {C_S} [/ latex] для последовательной емкости, примите во внимание, что

    [латекс] {V =} [/ latex] [latex] {\ frac {Q} {C_S}} [/ latex] [latex] {= V_1 + V_2 + V_3}. [/ Latex]

    Вводя выражения для [латекс] {V_1} [/ latex], [latex] {V_2} [/ latex] и [latex] {V_3} [/ latex], получаем

    [латекс] {\ frac {Q} {C_S} = \ frac {Q} {C_1} + \ frac {Q} {C_2} + \ frac {Q} {C_3}}.[/ латекс]

    Исключая [латекс] {Q} [/ latex] s, мы получаем уравнение для полной емкости в серии [латекс] {C_S} [/ latex], равное

    [латекс] {\ frac {1} {C_S}} [/ latex] [latex] {=} [/ latex] [latex] {\ frac {1} {C_1}} [/ latex] [latex] {+ } [/ latex] [латекс] {\ frac {1} {C_2}} [/ latex] [latex] {+} [/ latex] [latex] {\ frac {1} {C_3}} [/ latex] [ латекс] {+ \ cdots}, [/ latex]

    , где «…» означает, что выражение действительно для любого количества конденсаторов, подключенных последовательно. Выражение этой формы всегда приводит к общей емкости [латекс] {C_S} [/ латекс], которая меньше любой из отдельных емкостей [латекс] {C_1} [/ латекс], [латекс] {C_2} [/ латекс ],…, Как показано в следующем примере.

    Общая емкость в серии,

    C с

    Общая емкость в серии: [латекс] {\ frac {1} {C_S} = \ frac {1} {C_1} + \ frac {1} {C_2} + \ frac {1} {C_3} + \ cdots} [ / латекс]

    Пример 1: Что такое последовательная емкость?

    Найдите общую емкость для трех последовательно соединенных конденсаторов, учитывая, что их отдельные емкости составляют 1.000, 5.000 и 8.000 [латекс] \ mu \ text {F} [/ latex].

    Стратегия

    Имея данную информацию, общую емкость можно найти, используя уравнение для емкости в серии.

    Решение

    Ввод заданных емкостей в выражение для [латекс] {\ frac {1} {C_S}} [/ latex] дает [латекс] {\ frac {1} {C_S} = \ frac {1} {C_1} + \ гидроразрыв {1} {C_2} + \ frac {1} {C_3}} [/ латекс].

    [латекс] {\ frac {1} {C_S}} [/ latex] [latex] {=} [/ latex] [латекс] {\ frac {1} {1.000 \; \ mu \ text {F}}} [/ латекс] [латекс] {+} [/ латекс] [латекс] {\ frac {1} {5.000 \; \ text {F}}} [/ латекс] [латекс] {+} [/ латекс] [латекс ] {\ frac {1} {8.000 \; \ mu \ text {F}}} [/ latex] [латекс] {=} [/ latex] [латекс] {\ frac {1.325} {\ mu \ text {F}}} [/ латекс]

    Инвертирование для поиска [латекса] {C_S} [/ latex] дает [латекс] {C_S = \ frac {\ mu \ text {F}} {1.325} = 0.755 \; \ mu \ text {F}} [/ latex ].

    Обсуждение

    Общая последовательная емкость [латекс] {C_s} [/ латекс] меньше наименьшей индивидуальной емкости, как было обещано. При последовательном соединении конденсаторов сумма меньше деталей. На самом деле это меньше, чем у любого человека. Обратите внимание, что иногда возможно и более удобно решить уравнение, подобное приведенному выше, путем нахождения наименьшего общего знаменателя, который в данном случае (показаны только целочисленные вычисления) равен 40.Таким образом,

    [латекс] {\ frac {1} {C_S}} [/ latex] [latex] {=} [/ latex] [латекс] {\ frac {40} {40 \; \ mu \ text {F}}} [/ latex] [latex] {+} [/ latex] [latex] {\ frac {8} {40 \; \ mu \ text {F}}} [/ latex] [latex] {+} [/ latex] [латекс] {\ frac {5} {40 \; \ mu \ text {F}}} [/ латекс] [латекс] {=} [/ латекс] [латекс] {\ frac {53} {40 \; \ mu \ text {F}}}, [/ latex]

    так что

    [латекс] {C_S =} [/ латекс] [латекс] {\ frac {40 \; \ mu \ text {F}} {53}} [/ латекс] [латекс] {= 0,755 \; \ mu \ text {F}}. [/ latex]

    На рис. 2 (а) показано параллельное соединение трех конденсаторов с приложенным напряжением.Здесь общую емкость найти легче, чем в последовательном случае. Чтобы найти эквивалентную общую емкость [латекс] {\ text {C} _ {\ text {p}}} [/ latex], сначала отметим, что напряжение на каждом конденсаторе составляет [латекс] {V} [/ латекс], такие же, как и у источника, поскольку подключаются к нему напрямую через проводник. (Проводники являются эквипотенциальными, поэтому напряжение на конденсаторах такое же, как и на источнике напряжения.) Таким образом, конденсаторы имеют такой же заряд, как и при индивидуальном подключении к источнику напряжения.Общий заряд [латекс] {Q} [/ латекс] равен сумме индивидуальных сборов:

    [латекс] {Q = Q_1 + Q_2 + Q_3}. [/ Латекс]

    Рисунок 2. (а) Конденсаторы, включенные параллельно. Каждый из них подключен непосредственно к источнику напряжения, как если бы он был полностью один, поэтому общая параллельная емкость — это просто сумма отдельных емкостей. (b) Эквивалентный конденсатор имеет большую площадь пластины и поэтому может удерживать больше заряда, чем отдельные конденсаторы.

    Используя соотношение [латекс] {Q = CV} [/ латекс], мы видим, что общий заряд составляет [латекс] {Q = C _ {\ text {p}} V} [/ латекс], а индивидуальные расходы равны [латекс] {Q_1 = C_1 V} [/ латекс] , [латекс] {Q_2 = C_2 V} [/ латекс] , и [латекс] {Q_3 = C_3 V} [/ латекс].Ввод их в предыдущее уравнение дает

    [латекс] {C _ {\ text {p}} V = C_1 V + C_2 V + C_3 V}. [/ Латекс]

    Исключая [латекс] {V} [/ латекс] из уравнения, мы получаем уравнение для полной параллельной емкости [латекс] {C _ {\ text {p}}} [/ latex]:

    [латекс] {C _ {\ text {p}} = C_1 + C_2 + C_3 \ cdots} [/ латекс].

    Полная параллельная емкость — это просто сумма отдельных емкостей. (И снова «» указывает, что выражение действительно для любого количества конденсаторов, подключенных параллельно.) Так, например, если конденсаторы в приведенном выше примере были подключены параллельно, их емкость была бы

    [латекс] {C _ {\ text {p}} = 1.000 \; \ mu \ text {F} + 5.000 \; \ mu \ text {F} + 8.000 \; \ mu \ text {F} = 14.000 \; \ mu \ text {F}}. [/ latex]

    Эквивалентный конденсатор для параллельного соединения имеет значительно большую площадь пластины и, следовательно, большую емкость, как показано на Рисунке 2 (b).

    Общая емкость параллельно,

    C p [латекс] {C _ {\ text {p}}} [/ latex]

    Общая емкость параллельно [латекс] {C _ {\ text {p}} = C_1 + C_2 + C_3 + \ cdots} [/ latex]

    Более сложные соединения конденсаторов иногда могут быть последовательными и параллельными.(См. Рис. 3.) Чтобы найти общую емкость таких комбинаций, мы идентифицируем последовательные и параллельные части, вычисляем их емкости, а затем находим общую.

    Рисунок 3. (a) Эта схема содержит как последовательные, так и параллельные соединения конденсаторов. См. Пример 2 для расчета общей емкости цепи. (b) C 1 и C 2 идут последовательно; их эквивалентная емкость C S меньше, чем у любого из них.(c) Обратите внимание, что C S находится параллельно с C 3 . Таким образом, общая емкость равна сумме C S и C 3 .

    Смесь последовательной и параллельной емкости

    Найдите общую емкость комбинации конденсаторов, показанной на рисунке 3. Предположим, что емкости на рисунке 3 известны с точностью до трех десятичных знаков ([латекс] {C_1 = 1.000 \; \ mu \ text {F}} [/ latex], [латекс] {C_2 = 5.000 \; \ mu \ text {F}} [/ латекс] и [латекс] {C_3 = 8.000 \; \ mu \ text {F}} [/ latex]) и округлите ответ до трех десятичных знаков.

    Стратегия

    Чтобы найти общую емкость, мы сначала определяем, какие конденсаторы включены последовательно, а какие — параллельно. Конденсаторы [латекс] {C_1} [/ latex] и [латекс] {C_2} [/ latex] включены последовательно. Их комбинация, обозначенная на рисунке [латекс] {C_S} [/ latex], параллельна [латексу] {C_3} [/ latex].

    Решение

    Поскольку [latex] {C_1} [/ latex] и [latex] {C_2} [/ latex] включены последовательно, их общая емкость определяется как [latex] {\ frac {1} {C_S} = \ frac {1 } {C_1} + \ frac {1} {C_2} + \ frac {1} {C_3}} [/ latex].Ввод их значений в уравнение дает

    [латекс] {\ frac {1} {C_1}} [/ latex] [latex] {+} [/ latex] [latex] {\ frac {1} {C_2}} [/ latex] [latex] {= } [/ latex] [латекс] {\ frac {1} {1.000 \; \ mu \ text {F}}} [/ latex] [latex] {+} [/ latex] [latex] {\ frac {1} {5.000 \; \ mu \ text {F}}} [/ latex] [latex] {=} [/ latex] [latex] {\ frac {1.200} {\ mu \ text {F}}}. [/ Latex ]

    Инвертирование дает

    [латекс] {C _ {\ text {S}} = 0.833 \; \ mu \ text {F}}. [/ Latex]

    Эта эквивалентная последовательная емкость подключена параллельно третьему конденсатору; Таким образом, общая сумма составляет

    [латекс] \ begin {array} {r @ {{} = {}} l} {C _ {\ text {tot}}} \; = & {C_S + C_S} \\ [1em] = & {0.833 \; \ mu \ text {F} + 8.000 \; \ mu \ text {F}} \\ [1em] = & {8.833 \; \ mu \ text {F}}. \ end {array} [/ latex]

    Обсуждение

    Этот метод анализа комбинаций конденсаторов по частям, пока не будет получена общая сумма, может быть применен к большим комбинациям конденсаторов.

    • Общая емкость последовательно [латекс] {\ frac {1} {C _ {\ text {S}}} = \ frac {1} {C_1} + \ frac {1} {C_2} + \ frac {1} { C_3} + \ cdots} [/ латекс]
    • Общая емкость параллельно [латекс] {C _ {\ text {p}} = C_1 + C_2 + C_3 + \ cdots} [/ latex]
    • Если схема содержит комбинацию конденсаторов, включенных последовательно и параллельно, определите последовательную и параллельную части, вычислите их емкости, а затем найдите общую сумму.

    Концептуальные вопросы

    1: Если вы хотите хранить большое количество энергии в конденсаторной батарее, подключите ли вы конденсаторы последовательно или параллельно? Объяснять.

    Задачи и упражнения

    1: Найдите общую емкость комбинации конденсаторов на рисунке 4.

    Рисунок 4. Комбинация последовательного и параллельного подключения конденсаторов.

    2: Предположим, вам нужна конденсаторная батарея с общей емкостью 0.750 Ф, и у вас есть множество конденсаторов емкостью 1,50 мФ. Какое наименьшее число вы могли бы связать вместе, чтобы достичь своей цели, и как бы вы их связали?

    3: Какую общую емкость можно получить, соединив [латекс] {5.00 \; \ mu \ text {F}} [/ latex] и [латекс] {8.00 \; \ mu \ text {F}} [/ latex] конденсатор вместе?

    4: Найдите общую емкость комбинации конденсаторов, показанной на рисунке 5.

    Рисунок 5. Комбинация последовательного и параллельного подключения конденсаторов.

    5: Найдите общую емкость комбинации конденсаторов, показанной на рисунке 6.

    Рисунок 6. Комбинация последовательного и параллельного подключения конденсаторов.

    6: Необоснованные результаты

    (a) Конденсатор [латекс] {8.00 \; \ mu \ text {F}} [/ latex] подключен параллельно другому конденсатору, что дает общую емкость [латекс] {5.00 \; \ mu \ text { F}} [/ латекс]. Какая емкость у второго конденсатора? б) Что неразумного в этом результате? (c) Какие предположения необоснованны или непоследовательны?

    Решения

    Задачи и упражнения

    1: [латекс] {0.293 \; \ mu \ text {F}} [/ латекс]

    3: [латекс] {3.08 \; \ mu \ text {F}} [/ latex] в последовательной комбинации, [latex] {13.0 \; \ mu \ text {F}} [/ latex] в параллельной комбинации

    4: [латекс] {2.79 \; \ mu \ text {F}} [/ латекс]

    6: (a) [латекс] {- 3,00 \; \ mu \ text {F}} [/ латекс]

    (b) У вас не может быть отрицательного значения емкости.

    (c) Предположение, что конденсаторы были подключены параллельно, а не последовательно, было неверным.Параллельное соединение всегда дает большую емкость, в то время как здесь предполагалась меньшая емкость. Это могло произойти, только если конденсаторы подключены последовательно.

    КОНДЕНСАТОРЫ И ДИЭЛЕКТРИКИ

    КОНДЕНСАТОРЫ И ДИЭЛЕКТРИКИ

    Конденсатор — это набор проводников, который используется для хранения электрического заряжать. Очень простой конденсатор представляет собой изолированный металлический шар. Потенциал шара радиусом R и зарядом Q равен

    (27.1)

    Уравнение (27.1) показывает, что потенциал сферы пропорционален зарядить Q на проводнике. В целом это верно для любой конфигурации проводники. Это отношение можно записать как

    (27,2)

    где C называется емкостью системы проводов. Единица измерения емкости — фарад (Ф). Емкость металлический шар равен

    (27.3)

    Другой пример конденсатора — система, состоящая из двух параллельных металлических тарелки. В главе 26 было показано, что разность потенциалов между двумя пластины области A, расстояние разделения d, и с зарядами + Q и -Q, задается по

    (27,4)

    Используя определение емкости (уравнение (27.2)), емкость этого в системе можно рассчитать:

    (27.5)

    Уравнение (27.2) показывает, что заряд конденсатора пропорционален емкости C и потенциалу V. Для увеличения количества хранимого заряда на конденсаторе, сохраняя постоянный потенциал (напряжение), емкость конденсатор нужно будет увеличить. Поскольку емкость параллельного пластинчатый конденсатор пропорционален площади пластины А и обратно пропорционален на расстояние d между пластинами, этого можно добиться, увеличив площадь поверхности A и / или уменьшение разделительного расстояния d.Эти большие конденсаторы обычно изготавливаются из двух параллельных листов алюминизированной фольги, несколько дюймов в ширину и несколько метров в длину. Листы располагаются очень близко вместе, но удерживаемый от соприкосновения тонким листом пластика, зажатым между их. Весь бутерброд накрывают еще одним листом пластика и скручивают. вверх, как рулон туалетной бумаги.

    Пример: Задача 27.7

    Трубка счетчика Гейгера состоит из тонкой прямой проволоки. окружен коаксиальной проводящей оболочкой.Диаметр проволоки 0,0025 см, а раковины — 2,5 см. Длина трубки 10 см. Что такое емкость трубки счетчика Гейгера?

    Рисунок 27.1. Схема счетчика Гейгера.

    Задача будет решена в предположении, что электрическое поле генерируется бесконечно длинной линией заряда. Схематический вид сбоку трубы показан на рисунке 27.1. Радиус провода r w , радиус цилиндра r c , длина счетчика L, а заряд на проводе + Q.Электрическое поле в области между проволоку и цилиндр можно рассчитать по закону Гаусса. Электрическое поле в этой области будет иметь радиальное направление и его величина будет зависеть только от на радиальном расстоянии r. Рассмотрим цилиндр длиной L и радиусом r показано на рисунке 27.1. Электрический поток [Phi] через поверхность этого цилиндр равен

    (27,6)

    Согласно закону Гаусса, поток [Phi] равен вложенному заряду, разделенному Автор [epsilon] 0 .Следовательно,

    (27,7)

    Электрическое поле E (r) можно получить с помощью уравнения (27.7):

    (27,8)

    Разность потенциалов между проводом и цилиндром может быть получена следующим образом: интегрируя электрическое поле E (r):

    (27,9)

    Используя уравнение (27.2), можно рассчитать емкость трубки Гейгера:

    (27.10)

    Подставляя значения для r w , r c и L в ур.(27.10) получаем

    (27.11)

    Символ конденсатора показан на рисунке 27.2. Конденсаторы могут быть соединены вместе; они могут быть подключены последовательно или параллельно. Фигура 27.3 показаны два конденсатора емкостью C 1 и C 2 , подключены параллельно. Разность потенциалов на обоих конденсаторах должна быть равно и, следовательно,

    (27.12)

    Рисунок 27.2. Символ конденсатора. Рисунок 27.3. Два конденсатора подключены параллельно.

    Используя уравнение (27.12), можно рассчитать общий заряд обоих конденсаторов

    (27,13)

    Уравнение (27.13) показывает, что полный заряд конденсаторной системы, показанной на Рисунок 27.3 пропорционален разности потенциалов в системе. В два конденсатора на рисунке 27.3 можно рассматривать как один конденсатор с емкость C, где C относится к C 1 и C 2 в следующим образом

    (27.14)

    На рисунке 27.4 показаны два конденсатора емкостью C 1 и C 2 , соединены последовательно. Предположим, что разность потенциалов на C 1 составляет [Delta] V 1 , а разность потенциалов на C 2 представляет собой ΔV 2 . Заряд Q на верхней пластине вызовет заряд -Q на нижней пластине C 1 . Поскольку электрический заряд сохраняется, заряд на верхней пластине C 2 должен быть равен Q.Таким образом, заряд на нижней пластине C 2 равен -Q. В разность напряжений на C 1 определяется как

    (27,15)

    а разность напряжений на C 2 равна

    (27.16)

    Рисунок 27.4. Два конденсатора соединены последовательно. Дана общая разница напряжений на двух конденсаторах. по

    (27,17)

    Уравнение (27.17) снова показывает, что напряжение на двух конденсаторах, соединены последовательно, пропорциональна заряду Q. Система действует как одиночный конденсатор C, емкость которого может быть получена из следующих формула

    (27.18)

    Пример: Задача 27.10

    Многопластинчатый конденсатор, используемый, например, в радиоприемниках, состоит из четырех параллельные пластины, расположенные одна над другой, как показано на рисунке 27.5. Площадь каждой пластины — А, а расстояние между соседними пластинами — d.Что емкость этого устройства?

    Рисунок 27.5. Многопластинчатый конденсатор.

    Многоканальный конденсатор, показанный на рисунке 27.5, эквивалентен трем идентичным конденсаторы, подключенные параллельно (см. рисунок 27.6). Емкость каждого из три конденсатора равны и даются

    (27,19)

    Полная емкость многопластинчатого конденсатора может быть рассчитана с помощью уравнение (27.14):

    (27.20)

    Рисунок 27.6. Схема многопластинчатого конденсатора, показанного на рисунке 27.5.

    Три конденсатора, емкостью C 1 = 2,0 мкФ, C 2 = 5,0 мкФ и C 3 = 7,0 мкФ, первоначально заряжаются до 36 В подключив каждую на несколько мгновений к батарее на 36 В. Батарея тогда сняты и заряженные конденсаторы включены в замкнутую последовательную цепь, с соединением положительной и отрицательной клемм, как показано на Рисунке 27.7. Что будет окончательный заряд на каждом конденсаторе? Какое будет напряжение на точки PP ‘?

    Рисунок 27.7. Проблема 27.13.

    Начальные заряды на каждом из трех конденсаторов, q 1 , q 2 , q 3 , равны

    (27.21)

    После подключения трех конденсаторов заряд перераспределится. Заряды на трех конденсаторах после того, как система успокоится, равны Q 1 , Q 2 и Q 3 .Поскольку заряд сохраняется количество, существует связь между q 1 , q 2 и q 3 , и Q 1 , Q 2 , и Q 3 :

    (27,22)

    Напряжение между P и P ‘можно выразить через C 3 и Q 3 , или в терминах C 1 , C 2 , Q 1 и В 2 :

    (27,23)

    и

    (27.24)

    Используя уравнение (27.22), следующие выражения для Q 1 и Q 2 можно получить:

    (27,25)

    (27,26)

    Подставляя уравнение (27,25) и уравнение (27,26) в уравнение (27,24), получаем

    (27,27)

    Комбинируя уравнение (27.27) и уравнение (27.23), Q 3 можно выразить через известные переменные:

    (27.28)

    Подставляя известные значения емкости и начальных зарядов, мы получить

    (27,29)

    Напряжение на P и P ‘можно найти, объединив уравнения (27.29) и уравнение (27.23):

    (27.30)

    Заряды конденсатора 1 и конденсатора 2 равны

    . (27,31)

    (27,32)

    Если пространство между пластинами конденсатора заполнено изолятором, емкость конденсатора будет случайной по сравнению с ситуацией, в которой между пластинами есть вакуум.Изменение емкости вызвано изменение электрического поля между пластинами. Электрическое поле между пластины конденсатора будут создавать дипольные моменты в материале между тарелки. Эти наведенные дипольные моменты уменьшат электрическое поле в область между пластинами. Материал, в котором наведенный дипольный момент равен линейно пропорциональный приложенному электрическому полю называется линейным диэлектрик . В материалах этого типа полное электрическое поле между обкладки конденсатора E связаны с электрическим полем E free , которое существовал бы без диэлектрика:

    (27.33)

    где каппа называется диэлектрической проницаемостью. Поскольку финальный электрический поле E никогда не может превышать свободное электрическое поле E free , диэлектрическая проницаемость [каппа] должна быть больше 1.

    Разность потенциалов на конденсаторе пропорциональна электрическому поле между пластинами. Поскольку наличие диэлектрика снижает напряженности электрического поля, это также уменьшит разность потенциалов между обкладками конденсатора (если общий заряд на обкладках сохраняется постоянная):

    (27.34)

    Емкость C системы с диэлектриком обратно пропорциональна разность потенциалов между пластинами и связана с емкостью C освободить конденсатора без диэлектрика следующим образом

    (27,35)

    Поскольку [каппа] больше 1, емкость конденсатора может быть значительно увеличивается за счет заполнения пространства между обкладками конденсатора диэлектрик с большой каппа.

    Электрическое поле между двумя обкладками конденсатора представляет собой векторную сумму поля, создаваемые зарядами на конденсаторе, и поле, создаваемое поверхностные заряды на поверхности диэлектрика. Создаваемое электрическое поле зарядами на обкладках конденсатора (плотность заряда [sigma] бесплатно ) выдается

    (27,36)

    Полагая плотность заряда на поверхности диэлектрика равной [sigma] bound , поле, создаваемое этими связанными зарядами, равно к

    (27.37)

    Электрическое поле между пластинами равно E free / Каппа и таким образом

    (27,38)

    Подставляя уравнение (27,36) и уравнение (27,37) в уравнение (27,38), получаем

    (27,39)

    или

    (27,40)

    Пример: задача 27.19

    Конденсатор с параллельными пластинами с площадью пластин A и разделительным расстоянием d содержит пластину диэлектрика толщиной d / 2 (см. рисунок 27.8) и диэлектрический постоянная каппа. Разность потенциалов между пластинами составляет ΔV.

    а) Найдите электрическое поле в пустом пространстве по заданным величинам. область пространства между пластинами.

    б) Найдите электрическое поле внутри диэлектрика.

    в) Найдите плотность связанных зарядов на поверхности диэлектрика.

    Рисунок 27.8. Проблема 27.19.

    а) Предположим, что электрическое поле в конденсаторе без диэлектрика равно к E 0 .Электрическое поле в диэлектрике E d равно связано со свободным электрическим полем через диэлектрическую проницаемость [каппа]:

    (27,41)

    Разность потенциалов между пластинами может быть получена путем интегрирования электрическое поле между пластинами:

    (27,42)

    Таким образом, электрическое поле в пустой области равно

    (27,43)

    б) Электрическое поле в диэлектрике можно найти, комбинируя ур.(27,41) и (27.43):

    (27,44)

    c) Плотность свободного заряда [сигма] free равна

    (27,45)

    Плотность связанного заряда связана с плотностью свободного заряда через следующее отношение

    (27,46)

    Комбинируя уравнение (27.45) и уравнение (27.46), получаем

    (27,47)

    Электрическое поле в «пустом» конденсаторе можно получить с помощью закона Гаусса.Рассмотрим идеальный конденсатор (без краевых полей) и интегрирование объем, показанный на рисунке 27.9. Площадь каждой пластины конденсатора — А, а площадь заряды на пластинах +/- Q. Заряд, заключенный в объёме интеграции показанное на рисунке 27.9, равно + Q. Закон Гаусса гласит, что электрический поток [Phi] через поверхность объёма интегрирования относится к приложенному плата:

    (27,48)

    Если между пластинами вставлен диэлектрик, электрическое поле между пластинами пластины будут меняться (даже если заряд на пластинах остается постоянным).Очевидно, что закон Гаусса, сформулированный в уравнении (27.48), в этом случае не выполняется. Электрическое поле E между обкладками конденсатора связано с бездиэлектрическое поле E свободное :

    (27,49)

    где [каппа] — диэлектрическая проницаемость материала между пластинами. Теперь закон Гаусса можно переписать как

    (27,50)

    Закон Гаусса в вакууме является частным случаем уравнения (27.50) с [каппа] = 1.

    Рисунок 27.9. Идеальный конденсатор.

    Пример: задача 27.25

    Металлический шар радиуса R окружен концентрическим диэлектриком. оболочка с внутренним радиусом R и внешним радиусом 3R / 2. Это окружено концентрическая тонкая металлическая оболочка радиуса 2R (см. рисунок 27.10). В диэлектрическая проницаемость оболочки каппа. Какая у этого емкость хитрое изобретение?

    Предположим, что заряд на внутренней сфере составляет Q бесплатно .Электрический поле внутри диэлектрика можно определить, применив закон Гаусса для диэлектрика (уравнение (27.50)) и используя в качестве объема интегрирования сферу радиуса r (где R

    (27,51)

    Таким образом, электрическое поле в этой области определяется соотношением

    . (27.52)

    Рисунок 27.10. Проблема 27.25. Электрическое поле в области между 3R / 2 и 2R может быть полученный аналогичным образом, и равен

    (27.53)

    Используя электрическое поле из уравнений (27.52) и (27.53), мы можем определить разность потенциалов [Дельта] V между внутренней и внешней сферами:

    (27,54)

    Емкость системы может быть получена из уравнения (27.54) с использованием определение емкости через заряд Q и потенциал разница [Дельта] V:

    (27,55)

    Электрическая потенциальная энергия конденсатора, не содержащего диэлектрика и имеющего заряд +/- Q на его пластинах соответствует

    (27.56)

    где V 1 и V 2 — потенциалы двух пластин. Электрическая потенциальная энергия также может быть выражена через емкость С конденсатора

    (27,57)

    Эта формула верна и для конденсатора с диэлектриком; свойства диэлектрика входит в эту формулу через емкость C.

    Пример: Задача 27.40

    Десять одинаковых конденсаторов по 5 мкФ подключены параллельно к источнику питания 240 В. аккумулятор.Затем заряженные конденсаторы отключаются от аккумулятора и повторно подключены последовательно, положительный вывод каждого конденсатора подключается к отрицательной клемме следующего. В чем разница потенциалов между отрицательный вывод первого конденсатора и положительный вывод последний конденсатор? Если эти клеммы подключены через внешнюю цепь, как большая часть заряда будет течь по этой цепи при последовательном разряде. ? Сколько энергии выделяется при разряде? Сравните это обвинение и это энергия с зарядом и энергией, хранящейся в исходном, параллельном расположении, и объясните любые неточности.

    Заряд на каждом конденсаторе после подключения к батарее 240 В составляет равно

    (27,58)

    Разность потенциалов на каждом конденсаторе останется равной 240 В после конденсаторы включены последовательно. Общая разность потенциалов по десять конденсаторов, таким образом, равны

    (27,59)

    Если два концевых вывода конденсаторной сети соединены, заряд 1.2 мкКл будет течь от положительной клеммы к отрицательной (см. Рисунок 27.11).

    Рисунок 27.11. Проблема 27.40. Электрическая энергия, запасенная в конденсаторной сети перед разряд равен

    (27,60)

    Энергия, запасенная в каждом конденсаторе после заряда до 240 В, равна к

    (27,61)

    Очевидно, что при замене конденсатора энергия не теряется. конфигурация с параллельного на последовательный.

    Пример: задача 27.39

    Три конденсатора подключены, как показано на рисунке 27.12. Их емкости составляют C 1 = 2,0 мкФ, C 2 = 6,0 мкФ и C 3 = 8,0 мкФ. Если на два свободных клеммы, какой будет заряд на каждом конденсаторе? Что будет электрическая энергия каждого?

    Рисунок 27.12. проблема 27.39.

    Предположим, что напряжение на конденсаторе C 1 составляет В 1 , и Напряжение на конденсаторе (C 2 + C 3 ) составляет В 2 .Если заряд конденсатора C 1 равен Q 1 , тогда Заряд на параллельном конденсаторе также равен Q 1 . Потенциал разница по этой системе равна

    (27,62)

    Таким образом, заряд конденсатора 1 определяется разностью потенциалов [Дельта] V

    (27,63)

    Напряжение V 23 на конденсаторе (C 2 + C 3 ) связано с расходом Q 1

    (27.64)

    Заряд конденсатора С 2 равен

    (27,65)

    Заряд конденсатора С 3 равен

    . (27,66)

    Электрическая потенциальная энергия, запасенная в каждом конденсаторе, равна

    . (27,67)

    Для трех конденсаторов в этой задаче электрическая потенциальная энергия равна к

    (27.68)

    (27,69)

    (27,70)


    Отправляйте комментарии, вопросы и / или предложения по электронной почте на адрес [email protected] и / или посетите домашнюю страницу Фрэнка Вольфса.

    Лекция 8

    POP4 20,49
    Два конденсатора по 25 µ F и 5 µ F подключены параллельно с 100 В на каждом. Какая общая энергия хранится?
    A. 150 µ J
    Б.0,150 Дж
    С. 150 Дж
    D. 150 кДж
    Ответ

    POP4 20,49
    Два конденсатора по 25 µ F и 5 µ F подключены последовательно. Какой Δ В требуется для хранения 0,150 Дж?
    А. 22,8 В
    Б. 100 В
    С. 137 V
    D. 268 V
    Ответ

    klm Walker5e CnEx 21-19
    Переключатель на рисунке ниже изначально разомкнут, а конденсатор не заряжен. Если & Escr; = 6.00 В, R = 10,0 Ом и C = 72,0 µ F, какой ток протекает через батарею сразу после включения переключателя?
    схемотехнический симулятор

    А. 0,432 мА
    Б. 0,600 А
    С. 0,900 А
    D. 1.20 A
    Ответ

    klm Walker5e CnEx 21-19
    Переключатель на рисунке ниже изначально разомкнут, а конденсатор не заряжен. Если & Escr; = 6,00 В, R = 10,0 Ом и C = 72.0 µ F, какой ток течет через батарею долгое время после включения переключателя?

    А. 0,432 мА
    Б. 0,600 А
    С. 0,900 А
    D. 1.20 A
    Ответ

    klm Walker5e Ex 21-17
    Если C = 24,0 µ F в схеме ниже, какова эквивалентная емкость всей цепи?

    A. 8.00 µ F
    В. 16.0 µ F
    С. 36.0 µ F
    Д.72.0 µ F
    Ответ

    Walker5e 21,61
    Два конденсатора, C 1 = C и C 2 = 2 C , подключены к батарее. Конденсатор _____ накапливает больше энергии, когда они подключены к батарее последовательно, а конденсатор _____ накапливает больше энергии, когда они соединяются параллельно с батареей.
    А. С 1 С 1
    Б. К 2 К 1
    В. С 1 С 2
    D. C 2 C 2
    Ответ

    SJ6 28,33
    Батарея была подключена к схеме ниже в течение длительного времени. Какое напряжение на конденсаторе?
    А. 2,00 В
    Б. 4.00 В
    В. 6,00 В
    D. 8.00 V
    Ответ

    Б.0,150 Дж

    Д. 268 В


    D. 1.20 А

    Сразу после включения переключателя конденсатор разряжается и ведет себя как провод. Батарея 6,00 В пропускает 0,600 А тока через каждый из резисторов 10,0 Ом (или, если вы предпочитаете, два параллельных резистора имеют эквивалентное сопротивление 5,00 Ом), в результате чего через батарею протекает ток 1,20 А.


    Б.0,600 А

    Через долгое время после включения переключателя конденсатор полностью заряжен и ведет себя как разомкнутая цепь. Батарея на 6,00 В пропускает через левый резистор 10,0 Ом ток 0,600 А.


    C. 36.0 µ F

    Два последовательно соединенных конденсатора имеют эквивалентную емкость ½ C , и они соединены параллельно с третьим идентичным конденсатором. Следовательно, мы складываем две емкости, чтобы получить эквивалентную емкость 1.50 C , или 1,50 × 24,0 µ F = 36,0 µ F.

    C. C 1 C 2

    При последовательном соединении два конденсатора имеют одинаковый заряд. Заметив, что U = ½ Q ² / C , вы можете увидеть, что меньший конденсатор C 1 сохраняет больше всего энергии. (Убедитесь сами, что если напряжение батареи составляет 6,0 В и C = 10,0 µ F, C 1 накоплений 80 µ Дж и C 2 накоплений 40 µ Дж.

    Добавить комментарий

    Ваш адрес email не будет опубликован. Обязательные поля помечены *